Applied Mathematics
Applied Mathematics
Department of Mathematics
By Dagnaw Tantie
Unit one
Vectors
By Dagnaw Tantie
Theorem: If ⃗ is a vector in 2-space with initial point ( , ) and the terminal point
( , ), then ⃗=〈 − , − 〉. Similarly, If ⃗ is a vector in 3-space with initial
point ( , , ) and the terminal point ( , , ), then ⃗=〈 − , − , −
〉.
By Dagnaw Tantie
Note: − = + (− )
The definition of vector addition is illustrated in the following figures.
In general we can summarize arithmetic operations on vectors in 2-space and 3-space as follows:
S.N. Operations Let =〈 , 〉 and = Let =〈 , , 〉 and =〈 , , 〉
〈 , 〉 be vectors in be vectors in
By Dagnaw Tantie
1. Addition + =〈 + , + 〉 + =〈 + , + , + 〉
2. Scalar =〈 , 〉 =〈 , , 〉
Multiplication
3. Equality = if and only if = if and only if
= & = = , = & =
4. Negative − = 〈− ,− 〉 − = 〈− ,− ,− 〉
5. Subtraction − =〈 − , − 〉 − =〈 − , − , − 〉
6. Zero vector 0 = 〈0,0〉 0 = 〈0,0, 0〉
Properties of Vector Addition and Scalar Multiplication
Theorem: For any vectors u, v and w any scalars k and m, the following relationship holds:
1. + = + 5. k(mu) = (km)u
2. ( + ) = +( + ) 6. ( + )= +
3. +0=0+ = 7. (k + m)u = ku + mu
4. + (− ) = 0 8. 1u = u
1.3. Scalar/dot product
Definition: If = ( , , … ) and = ( , , … ) are vectors in n-space, then the
scalar product or dot product of u and v is written as . and is defined as . = +
+ …+ .
Note: . is not a vector. i.e. . is a scalar.
e.g. Let = (3,7,1), = (−8,2,4) and = (4, −2,5). Then
i . = (3,7,1). (−8,2,4) = 3 × −8 + 7 × 2 + 1 × 4 = −24 + 14 + 4 = −6.
ii . = (−8,2,4). (4, −2,5) = −8 × 4 + 2 × −2 + 4 × 5 = −32 − 4 + 20 = −16.
Ex: Let = (1,5, )& = (4, , ). If . = −2 then find the value of x.
Properties of Dot Product
Let u, v and w be vectors and ∈ .
1. . = . → Scalar product is commutative.
2. .( + ) = . + . → Scalar product is distributive over addition.
3. ( ). = ( . ) = . ( ).
4. . ≥ 0 and . = = 0 iff = 0.
Class Work:
By Dagnaw Tantie
1. Suppose and be two vectors in such that . = 12 then . = 12.
2. Let u and v be two vectors in . If . = 9, then (2 ). = 2( . ) = 18.
1.3.1. Magnitude of a Vector
Proof:
1. If = = (0,0, . . . ,0), then ‖ ‖ = √0 + 0 + ⋯ + 0 = 0. Suppose ≠ 0, such that
=( , , …, ). Now ≠ 0, for some . So that + + ⋯+ > 0 and hence
‖ ‖=√ . = + +⋯+ > 0.
Thus ‖ ‖ ≥ 0.
Exercises:
a) If the magnitude of the vector u is 6 then the magnitude of a vector -2u is 12.Because
‖−2 ‖ = |−2|‖ ‖ = 2‖ ‖ = 2 × 6 = 12.
b) If the magnitude of the vector u is 6 then the magnitude of a vector -u is also 12.Because
‖− ‖ = |−1|‖ ‖ = ‖ ‖ = 6.
c) Let = ( , 4) and ‖ ‖ = 5, find the values of x
Definition: The distance between two points ( , , …, ) and ( , , …, ) is given by
⃗ = ( − ) +( − ) +⋯+( − )
direction of A.
By Dagnaw Tantie
Normalizing of a Vector
The process of multiplying a vector v by the reciprocal of its length to obtain a unit vector with
the same direction is called normalizing v.
Definition: Let > . For a given vector v, the vector
1. =‖ ‖
=‖ ‖
is a unit vector with the same direction as v-the direction is the same
because ‖ ‖
is a positive scalar, and the length is 1 because ‖ ‖ = ‖ ‖
=
‖ ‖ = 1.
‖ ‖
2. = −‖ ‖
=‖ ‖
is a unit vector with opposite direction as v-the direction is opposite
because −‖ ‖
is negative scalar, and the length is 1 because ‖ ‖ = − ‖ ‖
=
‖ ‖= ‖ ‖ = 1.
‖ ‖ ‖ ‖
3. = .‖ ‖
= ‖ ‖
is a vector of magnitude k with the same direction as v-the
given).
4. = − .‖ ‖
=− ‖ ‖
is a vector of magnitude k with opposite direction as v-the
given).
Example: Let = + 3 − . Then find
1. the unit vector in the same direction of .
2. the unit vector in the opposite direction of .
3. the vector with magnitude 10 that has the same direction as .
4. the vector with magnitude 10 that has opposite direction as v.
Solution:
1. The vector v has length ‖ ‖ = 1 + 3 + (−1) = √11. So the unit vector u in the
same direction as v is = = + −
√ √ √ √
By Dagnaw Tantie
2. Since ‖ ‖ = √11, the unit vector u in opposite direction as v is =− =− −
√ √
+ .
√ √
3. Since ‖ ‖ = √11, the vector with magnitude 10 that has the same direction as is
= 10. = + − .
√ √ √ √
4. Since ‖ ‖ = √11, the vector with magnitude 10 that has opposite direction as is
= −10. =− − + .
√ √ √ √
In an −coordinate system the unit vectors along the x-,y- and z- axes are denoted by i and j,
respectively; and in an -coordinate system the unit vectors along line x-, y- and z- axes are
denoted by i. j, and k. respectively (see Figure below). Thus,
The vectors = 〈1,0〉, = 〈0,1〉 are in 2-space.
The vectors = 〈1,0,0〉, = 〈0,1,0〉 , = 〈0,1,0〉 arein 3-space.
Every vector in 2-space is expressible uniquely in terms of i and j; and every vector in 3-space is
expressible uniquely in terms of i, j, and k as follows:
=〈 , 〉 = 〈 , 0〉 + 〈0, 〉= 〈1,0〉 + 〈0,1〉 = +
=〈 , , 〉 = 〈 , 0,0〉 + 〈0, , 0〉 = 〈1,0,0〉 + 〈0,1,0〉 + 〈0,0,1〉 = + +
Example:
2-space 3-space
a 〈3,4〉 = 3 + 4 〈3,4,5〉 = 3 + 4 + 5
b 〈0,4〉 = 0 + 4 = 4 〈0,0,5〉 = 0 + 0 + 5 = 5
c (3 + 4 ) + (2 ) = (3 + 4 ) + (0 + 2 ) (4 + ) + ( + 3 ) = + 7 +
= 3 +6
d 3(3 + 4 ) = 9 + 12 2( + 4 − 3 ) − 3(− + +3 )
=5 − −9
By Dagnaw Tantie
e ‖3 + 4 ‖ = √3 + 4 = √25 = 5 ‖3 + 4 + 5 ‖ = √3 + 4 + 5 = 2√25
f ‖ + ‖= + ‖ + + ‖= + +
g (3 + 4 ). (−2 + 4 ) = 10 ( + 4 − 3 ). (− + +3 )=8
Perpendicular/Orthogonal Vectors
Definition: Two vectors u and v are said to be perpendicular or orthogonal if . = 0.
Example: 1) The vectors = 〈2, −2,0〉 and = 〈3,3,4〉 are orthogonal, because . =
〈2, −2,0〉. 〈3,3,4〉 = 0.
2) Find x so that the vectors = 〈 , −2,1〉 and = 〈 , , 1〉 are perpendicular.
Exercise:
1. Determine a scalar c so that the given vectors to be orthogonal.
a) =2 − +3 , =3 + +4
b) = 〈 , , 〉, = 〈−3,4, 〉.
By Dagnaw Tantie
The low of cosine applied to vectors.
.
Note: From equation (∗) we have cos =‖ ‖‖ ‖
Exercise:
a) Determine a scalar c so that the angle between = + and = + is 45 .
b) Suppose ‖ ‖ = 2 , ‖ ‖ = 2√7 and . = 2. Find the angle between and + .
1.3.3. Orthogonal projection
.
If the angle between u and v is acute, proj has length ‖ ‖ cos and direction is ‖ ‖
If the angle between u and v is obtuse, cos < 0 and proj has length −‖ ‖ cos and
.
direction is − ‖ ‖
By Dagnaw Tantie
. . .
Thus , proj = ‖ ‖ cos . ‖ ‖
= ‖ ‖.‖ ‖
=‖ ‖
and ‖ ‖ cos = ‖ ‖.
.
The number ‖ ‖ cos =‖ ‖
is called the scalar component of u in the direction of v and is
denoted by comp .
Now we can summarize this idea as follows:
. .
1. The vector projection of b on to a is proj = ‖ ‖.‖ ‖
=‖ ‖
.
.
2. The scalar projection or component of b on to a is comp = ‖ ‖ cos = ‖ ‖.
. ( ).( )
b) proj =‖ ‖
= = = = (2 + + 2 ).
√
. ( ).( )
c) comp =‖ ‖
= = .
( ) √
. ( ).( )
d) comp =‖ ‖
= = = 5.
√
Note:
1. proj ⊥ ( − proj ); i.e. (proj ) . ( − proj )
2. proj ∥ ; i.e. proj = , for some ∈ .
1.3.4. Direction Angles and Direction Cosines
By Dagnaw Tantie
Definition: The angles , & formed by the nonzero vector = + + and the positive
axis, respectively, are called direction angles. The cosine values cos = ‖ ‖, cos =
‖ ‖
& cos = ‖ ‖
of these angle are called direction cosines.
Or, equivalently
cosines of v.
cos =‖ ‖
= = ⇒ = cos ( ) ≈ 150 .
( ) √ √
cos =‖ ‖
= = ⇒ = cos ( ) ≈ 70 .
( ) √ √
By Dagnaw Tantie
cos =‖ ‖
= = ⇒ = cos ( ) ≈ 77 .
( ) √ √
So that the direction angles are 150 , 70 &77 . In additionally the direction cosines are
, & .
√ √ √
× = = − + .
Or equivalently
× =( − ) −( − ) +( − ) .
Note:
i. The cross product × of two vectors u and v, unlike the dot product, is a vector. For
this reason it is also called the vector product.
ii. Note that × is defined only when u and v are three-dimensional vectors.
Example: If =2 + − and = −3 + 4 + then
1 −1 2 −1 2 1
i. × = 2 1 −1 = − + = 5 − + 11 .
4 1 −3 1 −3 4
−3 4 1
4 1 −3 1 −3 4
ii. × = −3 4 1 = − + = −5 + − 11 .
1 −1 2 −1 2 1
2 1 −1
1 −1 2 −1 2 1
iii. × = 2 1 −1 = − + =0 +0 + .
1 −1 2 −1 2 1
2 1 −1
Remark:
× is orthogonal to both u and v.
× = 0 if and only if u and v are scalar multiple of each other(i.e. ∥ )
Example: Let =− + and = + 2 − . Find unit vectors which are orthogonal to both u
and v.
Solution: × and × are orthogonal to both u and v.
By Dagnaw Tantie
0 1 −1 1 −1 0
First let us determine, × = −1 0 1 = − +
2 −1 1 −1 1 2
1 2 −1
= −2 − 2 . and × =2 +2 .
Now we can find the a unit vectors in the direction of × and × .
×
‖ × ‖
= =− − &
√ √ √
×
‖ × ‖
= = + .
√ √ √
Thus, − − and + are unit vectors that are orthogonal to both u and v.
√ √ √ √
The following cross products occur so frequently that it is helpful to be familiar with them:
Theorem: Let u and v be two vectors and is the angle between u and v, then
‖ × ‖ = ‖ ‖‖ ‖ sin .
Example: Let = + + and = − . Find the sine of the angle between u and v.
Solution:
By Dagnaw Tantie
‖ ‖‖ ‖
‖ × ‖ = ‖ ‖‖ ‖ sin ⇒ sin = , but we have
‖ × ‖
2) If the vectors u, v and w do not lie in the same plane, then the surface area of the
parallelepiped with edges u, v and w is given by
. = 2| × | + 2| × | + 2| × |
3) If the vectors u, v and w do not lie in the same plane, then the volume of the
parallelepiped with edges u, v and w is given by
= | . ( × )|
By Dagnaw Tantie
Or, = (ℎ ℎ )( )
= ‖Proj × ‖‖ × ‖
( × ). ‖ × ‖
= ‖ × ‖
( × ) ‖ × ‖ = ‖(( × ). )( × )‖ ‖ × ‖
= ‖(( × ). )( × )‖ ‖ × ‖
= ‖( × ). ‖‖ × ‖ ‖ = ‖( × ). ‖
× ‖
Note:
1) The vector that lies in the same plane are said to be Coplanar.
2) . ( × ) = 0 if and only if , and are coplanar.
3)
Examples:
1. Find the area of the triangle determined by the points (1,1,1), (2,3,4) and
(3,0, −1).
2. Find the volume and surface area of the parallelepiped having = 3 −5 + , =2 −
2 and =3 + + as adjacent side.
3. Determine if the three vectors = 〈1,4, −7〉, = 〈2, −1,4〉 and = 〈0, −9,18〉 lies in
the same plane or not.
Solution:
1. The vectors ⃗ and ⃗ can be taken as two sides of the triangle.
Since ⃗ = + 2 + 3 and ⃗= −3 −5 .
We have ⃗× ⃗= 1 2 3 = − +8 −5 .
1 −3 −5
Thus, the area is = ‖− + 8 − 5 ‖ = √90 = √30.
By Dagnaw Tantie
3 −5 1
. ( × ) = 0 2 −2 = 36.
3 1 1
Thus the volume is, = | . ( × )| = |36| = 36.
3. If the volume is zero they lie in the same plane and if the volume isn't zero they don't lie in
the same plane.
1 4 −7
. ( × ) = 2 −1 4 = 0.
0 −9 18
So, the volume is zero and so they lie in the same plane.
Exercise: Find the volume of a parallelepiped (box) of determined by the vectors = +2 − ,
= −2 + 3 and =7 −4 .
1.5. Lines And planes
a) Paramedic equations of a line in space
Theorem: A line L parallel to the vector = 〈 , , 〉 and passing through the point ( , , )
is represented by the parametric equations
= + , y= + and = +
proof:
By Dagnaw Tantie
= +
Therefore, the equations = + are parametric equations of a line L passes through the
= +
point ( , , ) parallel to the vector 〈 , , 〉.
Note:
i) The vector V is a direction vector for the line L.
ii) The constants a, b and c are direction numbers.
iii) If the direction numbers a, b and c are all non zeros, we can eliminate the parameter t to
obtain symmetric equations of the line
= =
Example 1: Find the parametric and symmetric equations of the line L that passes through the
point (1, −2, 4) and parallel to = 〈2, 4, −4〉 .
Solution: we have (1, −2, 4) and = 〈2, 4, −4〉.
Therefore the parametric equations are
⇒ =1+2 , = −2 + 4 & =4−4
And the parametric equations are
= =
Example 2: Find the parametric equations for the line that passes through the points (4, 10, −6)
and (7, 9, 2).
Solution: Let = (4, 10, −6) and = (7, 9, 2). First we must find a direction vector for the line
passing through P and Q, given by
= ⃗ = 〈7 − 4, 9 − 10, 2 − (−6)〉 = 〈3, −1, 8〉 = 〈 , , 〉.
If we take =( , , ) = (4, 10, −6), the parametric equations are
⇒ = 4+3 , = 10 − & = −6 + 8
If we take =( , , ) = (7, 9, 2), the parametric equations are
⇒ = 7+3 , = 9− & =2+8
Note: The vector equation and parametric equation of a line is not unique. if we change the point
or the parameter or choose different parallel vectors, then the equations change.
Exercise: Find a set of parametric equations of a line that passes through the points (−2, 1, 0)
and (1, 3, 5); and list three points on L other than the given points.
b) Equations of planes
By Dagnaw Tantie
Theorem: The plane containing the point ( , , ) and having normal vector =〈 , , 〉
can be represented by the standard form of the equation of a plane,
( − )+ ( − )+ ( − )=0
Proof:
Since n is normal (perpendicular) to any vector that lies on the plane, we have
. ⃗=0
⇒ .〈 − , − , − 〉=0
⇒ 〈 , , 〉 .〈 − , − , − 〉=0
⇒ ( − )+ ( − )+ ( − ) = 0 ........which is an equation of the plane.
Note:
1. The general form of the equation of a plane in space is + + + = 0 , where
= −( + + ).
2. A vector perpendicular to the plane is called a norm to the plane.
3. If the vector 〈 , , 〉 is normal to the plane and 〈 , , 〉 be the vector les on the plane
, then + + = 0.
Example 1: Find the equation of the plane and x- intercept of the equation of the plane through
the point (1, 3, −1) with normal vector = 〈4, −1, 2〉.
Solution: An equation of the plane is
4( − 1 ) − 1( − 3) + 2( − (−1)) = 0
⇒4 − +2 +1 =0
To find the x-intercept of the plane we set = = 0.
Thus we obtain 4 + 1 = 0 ⇒ =− .
By Dagnaw Tantie
Example 2: Find the equation of the containing the points (2, 1, 1) , (0, 4, 1) & (−2, 1, 4) .
Solution: We need a point in the plane and a vector that is normal to the plane. There are three
choices for the point, but no normal vector is given. To obtain a normal vector, use the cross
product of vectors u and v extending from the point (2, 1, 1) to the points (0, 4, 1) and (−2, 1, 4).
The component forms of u and v are
= 〈0 − 2, 4 − 1, 1 − 1〉 = 〈−2, 3, 0〉 and = 〈−2 − 2, 1 − 1, 4 − 1〉 = 〈−4, 0, 3〉 .
If follows that
By Dagnaw Tantie
Solution: = 〈2, −1, 3〉 and = 〈−1, 5, 2〉. So that is neither ⊥ nor ∥ to , hence there is
an angle between and .
Now, . = 〈2, −1, 3〉 . 〈−1, 5, 2〉 = −2 − 5 + 6 = −1, ‖ ‖= 2 + (−1) + 3 = √14
: −2 = = = and : = = − 4 = , where , , ∈ .
Solution: : = 2+ , = −1 + 2 , = 3 + 3 and
: =5+3 , =1+2 , =4+ .
Thus solve,
2+ =5+3
.
−1 + 2 = 1 + 2
−3 =3
⇒ .
2 −2 =2
2 −6 =6
⇒ .
2 −2 =2
⇒ −4 = 4 ⇒ = −1 and = 0.
So that 3 + 3 = + 4 ⇒ 0 + 3 = −1 + 4 ⇒ 3 = 3.
Hence two lines are intersecting and the intersection point occurs when = 0.
= 2 + 0 = 2, = −1 + 2(0) = −1, = 3 + 3(0) = 3.
Therefore the point of intersection is = (2, −1, 3).
By Dagnaw Tantie
Note:
1. Two non- parallel and non-intersecting lines in space are called skew lines.
2. Two lines are parallel if the corresponding direction vectors are parallel.
Intersection of a line and a plane
If : = + , = + , = + be parametric equation of the line L and : +
+ + = 0 be the equation of the plan we can find the intersection point of L and by
substituting expression for x, y and z from the parametric equations in to the equation of the
plane. Next we solve for t and substitute back in to the parametric equations.
Example: find the point of intersection of the plane 3 − 2 = = −5 and the line =1+ ,
= −2 + 2 , =4 .
Solution: We substitute the expression for x, y, z from the parametric equations into the equation
of the plane.
3(1 + ) − 2(−2 + 2 ) + 4 = −5.
⇒ 3 + 3 + 4 − 4 + 4 = −5.
⇒ 7 + 3 = −5.
⇒ 7 + 3 = −5.
⇒ 3 = −12.
⇒ = −4.
Therefore the point of intersection occurs when the parameter value is = −4. Then =1+
(−4) = −3, = −2 + 2(−4) = −10, and = 4(−4) = −16.
So the point of intersection is (−3, −10, −16).
Distance between a point and a plane.
Theorem: The distance between a plane and a point (not in the plane) is
. ⃗
= Proj ⃗ = .
‖ ‖
By Dagnaw Tantie
Proof: Let ( , , ) be a point in the plane, and position the normal = ( , , ) so that its
initial point is at Q. As illustrated in the following figure, the distance D is equal to the length of
the orthogonal projection of ⃗ on .
⃗. . ⃗ . ⃗
= Proj ⃗ = = ‖ ‖= .
‖ ‖ ‖ ‖ ‖ ‖
But,
⃗=( − , − , − ).
⃗. =( − , − , − ). ( , , ) = ( − )+ ( − )+ ( − ).
‖ ‖=√ + + .
| ( ) ( ) ( )|
Thus, = √
| |
= √
|( ) ( )|
= ..................(∗)
√
Since the point ( , , ) lies in the plane, its coordinates satisfy the equation of the plane ;
that is,
+ + + =0
⇒ = −( + + ).
Substitute this expression in (∗) gives
| |
= .
√
Example: Find the distance between the point (1,5, −4) and the plane given by 3 − +2 =
6.
Solution: = 〈3, −1, 2〉 is normal to the given plane. To find a point in the plane, let = = 0,
and obtain the point (2,0,0). The vector from to p is given by
⃗ = 〈1 − 2, 5 − 0, −4 − 0〉 = 〈−1, 5, −4〉.
By Dagnaw Tantie
| ⃗. | |〈 , , 〉.〈 , , 〉|
Therefore, = ‖ ‖
= = .
( ) √
Where v is a direction vector for the line and is a point in the plane.
Example: Find the distance between the point (1, −2,0) and the line given by
: =2+3 , = −2 − 3 , = − .
Solution: First we find the point on the line, by setting = 0 we obtain (2, −2,0)
By Dagnaw Tantie
3. The elements 0 and 1 are also the elements of F.
Examples:
1. The set S = {(x, y, z): x, y, z ∈ R} is a vector space over the field R.
2. For any field F, the set of n-tuples is a vector space over F.
3. {0} is trivial vector space.
Vector Subspace
Definition: A non-empty subset W of a vector space V over a field F is called a vector subspace
or a linear subspace or simply a subspace of V if W is a vector space over F w.r.t. addition and
scalar multiplication.
Theorem: A non-empty subset W of V is a subspace of V iff
i) , ∈ ⟹ + ∈
ii) ,∈ , ∈ ⟹ ∈
Example 1: Let V= {( , , ): , , ∈ } and if = {( , , ): − 3 + 4 = 0} then prove
that W is a subspace of V.
Solution:
i) ≠ ∅, since 0 − 3(0) + 4(0) = 0, . This implies is non-empty subset of V.
ii) Let =( , , ) and =( , , ) be two vectors of W.
Then −3 +4 = 0 and −3 +4 =0
Adding these two equations yields
( + ) + 3( + ) + 4( + )=0
This implies + = ( + , + , + )∈
iii) Let =( , , )∈ and ∈ .
Since ∈ we have, −3 +4 = 0.
Thus multiply this equation both sides by C give
−3 +4 = 0.
This implies =( , , )∈
Hence W is a subspace of V.
By Dagnaw Tantie
But + = ( + , + , + )∉ as ( + ) + ( + ) ≠( + ) .
For instance, = (−3,4,5) and = (3,0,3) be two vectors of W as (−3) + 4 = 5 and
3 + 0 = 3 . But + = (0,4,8) ∉ as 0 + 4 ≠ 8 .
Example 3: The set = with standard vector addition and multiplication defined as
( , )=( , ) is not a vector space. Because, for any two scalars , ∈ , and
=( , ) ∈ ,( + ) ≠ + as
( + ) =( + )( , ) = ( ,( + ) )=( , + ) and
+ = ( , )+ ( , )=( , )+( , ) = (2 , + ).
This shows that ( + ) ≠ + and hence it is not a vector space.
Example 4: The set = with standard vector addition and multiplication defined as
( , , ) = (0,0, ) is not a vector space.
For instance, 1 = 1( , , ) = (0,0, (1) ) = (0,0, )≠( , , )= .
Example: Let W be the set of all points ( , ), from in which ≥ 0. Is this a subset of ?
Solution:
i. Let ( , )∈ and ( , )∈ . Then ( , )+( , )=( + , + )∈ ,
since , ≥ 0, we have + ≥ 0.
ii. Let ( , )∈ and ∈ .Now, ( , )= ( , )∉ , because is always
non-negative.
Exercises:
By Dagnaw Tantie
a) Linear combinations
Definition: Let V be a vector space over a field F and v , v , … , v be elements of V. let
α ,α ,…,α ∈ . An expression of the form α v + α v + α v + ⋯ + α v is called a linear
combination of v , v , … , v .
i.e. Let V be a vector space and v , v , … , v be elements of V. A vector ∈ is called a
linear combination of the vectors v , v , … , v if it can be written in the form
w = α v +α v +α v +⋯+ α v for some scalars α , α , … , α
Example 1: write the vector v =(6,-11) as a linear combination of vectors v = (2,3) and
v = (4,2) in a vector space V over a field R.
Solution: Let v = α v + α v , where α , α ∈ .
(6,-11)= α (2,3) + α (4,2)
(6,-11)= (2α , 3α ) + (4α , 2α )
(6,-11)= (2α + 4α , 3α + 2α )
2α + 4α =6
3α + 2α = −11
Solving these system of equation gives α = −7 and α = 5.
Hence the linear combination is (6,-11) =−7(2,3) + 5(4,2).
Exercise:
1. Express the vector (7,11) as a linear combination of the vectors (2,3) and (3,5).
2. Express the vector (15, 18) as a linear combination of the vectors (3, 4) and (3,3) .
3. Write the vector = (1,7, −4) as a linear combination of the vectors = (1, −3,2)
and = (2, −1,1) in a vector space V over a field R. Answer:(1,7, −4) = −3 +2 .
4. Write the polynomial = + 4 − 3 over R as a linear combination of polynomials
= − 2 + 5, =2 − 3 and = + 3. Answer: = −3 +2 +4 .
b) Generators
Definition: Let S be a non-empty subset of a vector space V over a field F. Then S is said to be
the generator (span) of V if each elements of V can be expressed as a linear combination of the
elements of S. Thus, if S is the generator of the vector space V and if v ∈ V, then there
exists v , v , … , v ∈ S such that V = α v + α v + α v + ⋯ + α v for α ∈ F
Example 1: Let V = R2 over R and S ={(1, 0)(0, 1)}. Show that S is a generator of V = R2.
Solution: To show this let =( , )∈R .
By Dagnaw Tantie
Then (x, y) = α (1, 0) + α (0, 1) ; α , α ∈ R
⇒ (x, y) = ( α , α )
⇒ α = x and α = y, such that
⇒ = (x, y) = x(1, 0) + y(0, 1)
Hence S is a generator of V.
Example 2: Show that the vectors v = (1, 2, 3), v = (0, 1, 2) and v = (0, 0, 1) generates
= over R.
Solution: To show this let =( , , )∈ = R and , , ∈ then
( , , )= v + v + v
⟹( , , )= (1, 2, 3) + (0, 1, 2) + (0, 0, 1)
⟹( , , )=( ,2 ,3 ) + (0, ,2 ) + (0, 0, )
⟹ ( , , ) = ( ,2 + ,3 +2 + )
=
⟹ 2 + =
3 +2 + =
Clearly = ∈ , since( , , ) ∈ =R , ∈ . Substitute x in place of in the
second equation implies = − 2 ∈ , since , ∈ .Also substitute x, and y-2x in
place of and in the second equation implies = −3 − 2( − 2 ) = −2 ∈ .
∴ ( , , ) = (1, 2, 3) + ( − 2 )(0, 1, 2) + ( − 2 )(0, 0, 1) .
Hence the vectors v , v and v generates =R .
Linear dependence and independence of vectors
a) Linear Dependence of Vector
Definition: Let V be a vector space over a field F and let v , v , … , v be elements of V. we say
v , v , … , v are linearly dependent over F if there exists α , α , … , α ∈ F (not all zero) such that
α v + α v + α v + ⋯ + α v = 0.
By Dagnaw Tantie
⟹ (3α + 2α , α + 2α − 4α , −4α − 3α + α ) = 0
3α + 2α = 0
⟹ α + 2α − 4α = 0 … … … … … … … … … … . (∗)
−4α − 3α + α = 0
Solving the last two equations (multiply the third equation by 4 and add with the second
equation) implies
3α + 2α = 0
⟹
−15α − 10α = 0
⟹ 3α + 2α = 0, since the second equation is -5 times the first equation.
⟹α = α … … … … … … … … … … . (∗∗)
Hence the solution set of equation (∗) using equations (∗∗) and (∗∗∗) is the set α ,α ,
:α ∈ .
By Dagnaw Tantie
b) Linear Independence of Vector
Definition: If V is a vector space over a field F then the vectors v , v , … , v are said to be linear
independent (LI) if there exist α , α , … , α ∈ F (all zero) such that α v + α v + α v + ⋯ +
α v = 0.
Exercise: show that the vectors (2, 4, 0), (0, 1, 0), (2, 6, 2) are linearly independent in the real
vector space .
Exercises:
1. Show that the vectors (1,0,0), (0,1,1) and (1,1,1) in the real vector space are linearly
independent.
2. Determine k so that the vectors (1, −1, 2), (0, , 3) and (−1,2,3) are linearly dependent.
3. Show that the vectors (2,1,0), (1, 1,0), (4,2,0) of are not linearly independent .
Solution 1: The value of the determinant form by the given vectors is
1 0 0
1 1
∆= 0 1 1 = = 1 − 1 = 0.
1 1
1 1 1
The determinant value is zero, therefore the given vectors are linearly dependent.
Solution 2: Since the vectors are linearly dependent
By Dagnaw Tantie
1 −1 2
3 0 3 0
0 3 =0 ⇒ + + =0
2 3 −1 3 −1 2
−1 2 3
⇒ 3 −6+3+ =0
⇒ 4 −3=0 ⇒ = .
2 1 0
Solution 3: 1 1 0 = 0. Hence the vectors are not linearly independent.
4 2 0
Bases and dimension of a vector space
Definition: Let V be a vector space over a field F. The set {v , v , … , v } of n-vectors of V forms
a base of V if
i) {v , v , … , v } span V and
ii) {v , v , … , v } is linearly independent.
Example: Let = and = {(2, 0, 0), (0, 9, 0), (0, 0, 5)}. Show that S is a basis of = .
Solution: First let us show S is the span of V. Now let ( , , ) ∈ = and , , ∈ , then
( , , ) = (2, 0, 0) + (0, 9, 0) + (0, 0, 5)
⟹ ( , , ) = (2 , 9 , 5 )
⟹ = , = , and = .
By Dagnaw Tantie
1. let = then the dim = 2. Since the standard basis of V are (1, 0) and (0, 1).
2. dim( )= .
3. dim( × ( )) = .
4. dim{0} = 0, since {0} has no basis.
Exercise: Given that = {( , , ) ∈ : 3 − 4 + = 0} is a subspace in . Find a basis and
the dimension of S.
Solution: Let = ( , , ) be a vector of S. Then 3 − 4 + = 0. . . = −3 + 4 .
Now, = ( , , ) = ( , , −3 + 4 )
= ( , 0, −3 ) + (0, , 4 )
= (1, 0, −3) + (0, 1, 4)
Thus the vector X is a linear combination of the vectors (1, 0, −3) and (0, 1, 4). Therefore,
= {(1, 0, −3), (0, 1, 4)} generates S. Also the vectors (1, 0, −3) and (0, 1, 4) are linearly
independent. Hence w is a basis of S. The number of vectors of w is two, the dimension of S is 2.
By Dagnaw Tantie
2. The matrix A is the order of this matrix is × (m by n).
3. We can abbreviate A by writing = .
×
1 4
Example: Let = 2 5 . Then
3 6
i) A matrix A has 3-rows and 2-columns.
ii) The order of the matrix A is 3 × 2.
iii) The elements of a matrix A are = 1, = 2, = 6, etc.
iv) The rows of a matrix A are [1 4], [2 5] and [3 6].
1 4
v) The columns of A matrix A are 2 and 5 .
3 6
Matrix Operation (Algebra of Matrix)
i) Equality of Matrices
Definition: Two matrices A and B are equal if they are the same order and if their corresponding
elements are identical (i.e. = ).
Examples:
1 2 1 2
1. If = and = then A=B. Since both A and B are order 2 × 2 and since
3 4 3 4
each element of is equal to corresponding element of B.
1 9 1 9 5 the A=B if and only if a=5 and b=-3.
2. If = and =
−3 4 7 4 7
ii) Addition of Matrices
Definition: Let = and = , the + = + .
× × ×
Note: The addition of matrices A and B is only defined if the matrices each have the same
number of rows and the same number of columns.
4 9 1 0 9 5
Example: Let = and = then
−3 4 7 7 3 1
4+0 9+9 1+5 4 18 6
i) + = = .
−3 + 7 4 + 3 7 + 1 4 7 8
0+4 9+9 5+1 4 18 6
ii) + = =
7 + (−3) 3 + 4 1 + 7 4 7 8
Remark: Addition of matrices is both commutative both commutative and associates.
iii) Multiplication of Matrix by a Scalar
By Dagnaw Tantie
Definition: Let = be a matrix and be a scalar (real or complex). Then we define
×
13 9 39 27
Example: Let = then 3 = .
−6 4 −18 12
Properties of scalar multiplication of matrices
Let A, B be to × matrices over a field F and , ∈ . Then
i) -1.A=A
ii) ( + )A= +
iii) ( + )= +
iv) ( )=( )
4 8 15 −9
Exercise: Let = and = . Then find
−6 2 3 8
i) 2A-B iii) 3(4A)
ii) 3A+B-2A iv) 3A+3B
2.2.Product of matrices and some algebraic properties; transpose of a matrix
a) Multiplication of Matrices
Definition: Let = and = then =[ ] × , where =
× ×
∑ = + +⋯+ .
Note: The multiplication of two matrices can be carried out only if the number of columns of the
first matrix equals the number of rows of the second matrix. Such a matrix is said to be
comfortable for multiplication.
−1 5 2 3 −6
Example: Let = and = . Then
7 −3 4 −8 9
−1.2 + 5.4 −1.3 + 5. −8 −1. −6 + 5.9 18 −43 51 .
i) AB= =
7.2 − 3.4 7.3 − 3. −8 7. −6 − 3.9 2 45 −69
ii) BA does not exist, since the first matrix B has 3-columns and the second matrix A has
2-rows.
Class work:
3 4
a) Find if = .
−2 5
1 7 −6
b) Can we find if = ?
4 6 9
By Dagnaw Tantie
Some properties of matrix multiplication
1) Matrix multiplication is not commutative.
2) Matrix multiplication is associative.
3) Matrix multiplication is distributive w.r.t addition.
b) Transpose of Matrix.
Definition: Let A= be to × matrix. Then the transpose of A, denoted by , is the
matrix obtained from A by interchanging rows and columns to produce the × matrx.
= =
12
−5 7
eg : If = then
9
1 4
12 9
i) = −5 1
7 4
12 −5 7
ii) ( ) = =
9 1 4
36 27
iii) (3 ) = −15 3
21 12
7
eg : If = [7 3 2] then = 3 .
2
Properties of transpose of matrices
Let A, B be to × matrices over a field F and ∈ . Then
i) ( ) = iii) ( ) =
ii) ( + ) =( + ) iv) ( ) =
By Dagnaw Tantie
7
eg: = 3 is a column matrix of order 3 × 1.
2
3. Zero or Null matrix: A matrix each of those elements s zero is called a null or zero
matrix.
0 0 0
eg: = is a 2 × 3 null matrix.
0 0 0
4. Square Matrix: A matrix with the same number of number of rows and columns (m=n)
is called a square matrix.
5. Diagonal matrix: A square matrix whose every element other than the diagonal element
is zero is called a diagonal matrix.
i.e. If all of the elements of a square matrix are zero except those in leading diagonal,
then the matrix is called as a diagonal matrix.
6. Unit Or identity matrix; A diagonal matrix with all its diagonal elements equal to 1 (i.e.
= 1 or ≠0 ≠ ) is called a unit or identity matrix.
1 0 0
Eg. 0 1 0 is a unit matrix of order 3.
0 0 0
7. Scalar Matrix: A diagonal matrix of order n in which all diagonal elements are equal, is
called scalar matrix.
3 0
Eg. is a scalar matrix of order 2.
0 3
8. Triangular Matrix: A square matrix in which all elements either below or above the
principal diagonal are zero is called triangular matrix.
I) Upper Triangular Matrix: A square matrix in which all elements below the
leading diagonal is equal to zero is called upper triangular matrix. Thus square
matrix A= is upper triangular matrix if =0 > .
1 2 3
Eg. 0 3 2 is an upper triangular matrix.
0 0 4
II) Lower Triangular Matrix: A square matrix in which all elements above the
leading diagonal is equal to zero is called lower triangular matrix. Thus square
matrix A= is lower triangular matrix if =0 > .
By Dagnaw Tantie
5 0 0
Eg. 3 3 0 is an lower triangular matrix.
9 8 4
9. Symmetric Matrix: A square matrix A= is said to be symmetric if it is equal to its
transpose matrix = .
i.e. A symmetric matrix is square matrix where = for all elements.
5 3 9
Eg. = 3 7 8 is symmetric matrix, since = = .
9 8 4
10. Skew- symmetric matrix: A square matrix A= is called skew-symmetric if
=− .
0 3 9
eg: = −3 0 8 is symmetric matrix, since =− .
−9 −8 0
Theorem: Let A be a square matrix, then
i) + is symmetric.
ii) − is skew- symmetric.
Remark :
a) If A be any square matrix, then and are symmetric.
b) If A and B both are symmetric, them AB is symmetric iff A and B are commutative.
c) Every square matrix can be uniquely expressed as the sum of a symmetric and skew
symmetric matrix. i.e. = ( + )+ ( − ).
By Dagnaw Tantie
Definition: Two × matrices said to be row equivalent if one can be obtained from the other
by means of a sequence of elementary row operations. Row equivalence between matrices A and
B is denoted by writing ~ .
1 2 0 1 2 0 1 2 0
Example: = 2 1 1 → +2 4 −1 5 ↔ 1 −1 2 →
1 −1 2 1 −1 2 4 −1 5
2 4 0
1 −1 2 = .
4 −1 5
Thus A is row- equivalent to B. i.e. ~ .
Elementary matrix
Definition: An × elementary matrix is any matrix that is obtained form an × unit matrix
I by performing a single elementary row operation.
1 0 0 1 0 0
i) 0 1 0 iii) 0 1 0
0 0 1 0 0 4
1 0 0 1 0 0
ii) 0 0 1 iv) 0 1 0
0 1 0 5 0 1
Echelon and row-reduced echelon forms of a matrix
i) The first non-zero elements in each row, is called its leading entry, is 1.
ii) In any two successive rows and + 1 that do not consist entirely of zeros the
leading element in the ( + 1) row lies to the right of the leading element in
row.
i.e. if two successive rows are non-zero, the second row starts with more zeros, than
the first (moving from left to right).
iii) Any row that consists entirely of zeros lie at the bottom of the matrix.
Example; Each of the following matrices shown below are examples of matrices in row echelon-
form.
1 2 3 4
1 2 3 4 1 2
0 0 1 3 and 0
0 0 1 3 , 0 1.
0 0 1
0 0 0 1 0 0 0
0 0 0
By Dagnaw Tantie
Definition: A matrix s to be row-reduced echelon form when it satisfies the following conditions.
1 2 0 0
1 2 0 0 1 0
Example: 0 0 0 1 0 and 0
0 1 1 , 0 1 are row- reduced echelon form.
0 0 1
0 0 0 1 0 0 0
0 0 0
Exercise: Reduce in to row- reduced echelon form using elementary row operation.
1 0 −2 1 1 5 2 1
a) 2 −1 0 1 and b) 5 3 −1 1
3 6 2 0 1 0 5 1
Rank of a matrix using elementary row operations
Definition: let A be m × matrix and let A be the row echelon form of a matrix A. The rank of
a matrix A is the number of non-zero rows of A and denoted by ( ).
1 0 −1 0
Example: 1. let = 0 1 1 1 then ( ) =2, because the number of non-zero rows of
0 0 0 0
0 0 0 0
the row echelon form of a matrix A is 2.
1 2
2. Find the rank of a matrix A= 2 4 .
−3 −6
1 2 1 2
→
Solution: A= 2 4 →
0 0
−3 −6 0 0
∴ ( ) =1
Exercise: Find the rank of a matrix A, if
1 2 3 2 1 6 6
1 2 4 A= 3 1
i) A= 1 1
ii)
−2 −4 −9 5 2 7 2
−3 4 3 2
System of linear equations and method of solving
Definition: A system of linear equations is a collection of m equations with n
a x + a x +⋯ +a x =b
a x + a x +⋯ +a x =b
unknowns x , x , … , x , .
⋮ ⋮
a x + a x +⋯ +a x =b
By Dagnaw Tantie
a a … a b x
a a … a b x
We can write as Ax = b, where A = ⋮ ⋮ ⋮ ⋮ ,b= and X = ⋮ .
⋮
a a a a b x
0
A is called the coefficient. If b = 0 the system is called homogenous
⋮
0
Theorem: Every system of linear equations has no solutions, or has exactly one solution, or has
infinitely many solutions.
Theorem (The rank test for number of solutions):Suppose = is a linear equation where A is
an × matrix. Then,
1) If ( | )= ( ) = , then the system has a unique solution and the system is
consistent and independent.
2) If ( | )= ( )= < , then the system has more than one solution and the
system is consistent and dependent.
3) If ( | )≠ ( ), then the system has no solution and the system is consistent.
THEOREM(Equivalent Statements): If A is an matrix, then the following are equivalent.
a) A is invertible.
b) = 0 has only the trivial solution.
c) The reduced row-echelon form of A is .
d) A is expressible as a product of elementary matrices.
e) = is consistent for every × 1matrix b.
f) = has exactly one solution for every × 1 matrix b
To solve the system of linear equation
i) Write in the form AX=b
ii) B= (A|b)
iii) Apply elementary operations on a matrix (A|b) to reduce row echelon form.
iv) Use back substitution
−2 +3 = 9
Example: Solve − + 3 = −4
2 − 5 + 5 = 17
Solution:
By Dagnaw Tantie
1 −2 3 9 x
i) AX=b, where A= −1 3 0 , b= −4 and X= x
2 −5 5 17 x
1 −2 3 9
ii) ( | )
B= A b = −1 3 0 −4
2 −5 5 17
1 −2 3 9 1 −2 3 9
→
iii) B= −1 3 0 −4 0 1 3 5
→
2 −5 5 17 0 −1 −1 −1
1 −2 39 1 −2 3 9
→ + 0 1 35 → 0 1 35
0 0 24 0 0 12
iv) Back substitution
Now, 0x1+0x2+x3 =2 ⇒ x3=2
0x1+x2+3x3= 5 ⇒ x2+3(2)= 5 ⇒ x2= −1
x1-2x2+3x3 = 9 ⇒ x1-2(-1)+3(2)= 9 ⇒ x1 = 1
Thus the solution is = 1, = −1, = 2.
Exercise: solve
+2 + =7 2 −3 + 3 = −8
a) + + − =3 b) 3 +2 + 2 = 10
3 + +5 −7 =1 − −2 =1
− + 5 + 2 =4
Individual assignment: solve 4 + 3 + 6 = 8 .
−2 + 6 + 4 = 12
Inverse of a matrix using elementary row operations
Definition: If A is a square matrix and we can find another matrix of the same size, say B, such
that
AB = BA = I
then we call A invertible and we say that B is an inverse of the matrix A. If we can’t find such a
matrix B we call A a singular matrix.
Procedures for computing the inverse of square matrix A;
i) Write the augmented matrix B= (A|I).
ii) Use elementary row operations to reduce the matrix A to its reduced row echelon
form.
By Dagnaw Tantie
iii) Deduce whether A is invertible or not. i.e.;
a) If A can be reduced to the identity matrix I, then A will be the matrix to the
right of the vertical bar.
b) If the row reduction does not give the identity matrix, then A is not invertible.
Properties of inverse of a matrix
Let A, B, C be invertible matrices of order n, then
i) (A ) = (A ) iii) (AB) =B A
ii) (A ) =A iv) (ABC) =C B A
Note: A A= AA =I
1 4 1 0 1 0
R → R R → R − 4R
0 1 0 1
2 −2 4
Solution: ii) Given A= 2 3 2
−1 1 −1
2 −2 4 1 0 0 1 −1 2 0 0
We can write, B= (A|I) = 2 3 2 0 1 0 R → R 2 3 2 0 1 0
−1 1 −1 0 0 1 −1 1 −1 0 0 1
0 0 0 0
1 −1 2 1 −1 2
→ ⎛ ⎞
→
0 5 −2 −1 1 0 R → R ⎜0 1 − 0⎟
0 0 1 0 1 0 0 1 0 1
⎝ ⎠
By Dagnaw Tantie
1 0 0 − −
⎛ ⎞ → ⎛1 0 0 ⎞
R → R + R ⎜0 1 − 0⎟ ⎜0 1 0 0 ⎟
→
0 0 1
0 0 1 0 1 0 1
⎝ ⎠ ⎝ ⎠
− −
⎛ ⎞
Thus, A =⎜ 0 ⎟
0 1
⎝ ⎠
− −
2 −2 4 ⎛ ⎞ 1 0 0
Check: AA = 2 3 2 ⎜0 ⎟= 0 1 0 =I
−1 1 −1 0 0 1
0 1
⎝ ⎠
Remark: Suppose A= . Then show that A is invertible if and only if ad-bc ≠0.
−
Show that when A is invertible, then A = .
−
Exercise: Find the inverse of a matrix A, if
2 −1
i) A=
3 6
1 −3 2
ii) = 2 0 0
1 4 1
By Dagnaw Tantie
Unit Four: Determinants
4.1.Definition of a determinant
a a … a
… a
⎛a a ⎞
Definition: If A = ⎜a a … a
⎟, we define determinant of A, (also denoted by detA),
⋮ ⋮ ⋮ ⋮
⎝a a … a ⎠
to be the scalar, det A =∑ (−1) a det A , for 1≤ i ≤ n.where A is a matrix which is
obtained by deleting the i row and j column of the matrix A.
Note:
1. If A = ( ) then det A = .
2. If A = then det A = . − .
a) A = (5)
1 2
b) A =
3 4
3 1 0
c) A = 2 −3 1
1 −1 4
Solution:
a) det A = 5
b) det A = 1 × 4 − 3 × 2 = −2.
c) Choose = 1
d) det A = ∑ (−1) a det A ,
= (−1) a det A + (−1) a det A + (−1) a det A
By Dagnaw Tantie
−3 1 2 1 2 −3
= 3 − +0 = 3(−12 + 1) − 1(8 − 1) + 0 = −40
−1 4 1 4 1 −1
15 −21
a) A =
−8 7
1 2 0
b) A = −3 2 1
1 −1 −5
−1 3 6 2
c) A = 2 0 5 7
2 1 3 3
4 5 6 −6
1. det(A , A , … , A ) =det(A , A , … A + A … A )
= det(A , A , … A … A ) + det(A , A , … A … A )
15 −21 7+8 −11 − 10
Eg. Let A = . Since A = , det(A) =
−8 7 −10 + 2 2+5
7 −11 8 −10
det + det .
−10 2 2 5
2. det(A , A , … λA … A ) = λ det(A , A , … A … A )
1 2 0 1 1 0
Eg. −3 2 1 =2 −3 1 1
1 −4 −5 1 −2 −5
3. If two columns are interchange then the determinant changes by sign.
7 −11 −11 7
Eg: det = det .
−10 2 2 −10
4. If one column is a scalar multiple of the other column then the determinant is zero.
1 2 3
Eg: −3 −6 1 = 0. Because column is a scalar multiple of column 1.
1 2 −5
5. If one of the columns is zero then the determinant is zero.
1 0 3
Eg: −3 0 1 =0
1 0 −5
By Dagnaw Tantie
6. If one adds a scalar multiple of one column to another then the determinant doesn’t
change.
3 0 1 3 2(3) + 0 1 3 6 1
Eg: 1 2 5 = 1 2(1) + 2 5 = 1 4 5
−1 4 2 −1 2(−1) + 4 2 −1 2 2
7. Let A = a be a diagonal matrix, then det A= a a ⋯a .
×
2 0 0 0
Eg. A = 0 3 0 0 = 2 × 3 × 3 × −6 = −108
0 0 3 0
0 0 0 −6
8. det A = det A .
7 −11 7 −10
Eg: det = det .
−10 2 −11 2
9. det (A B) = det A . det B
Eg: If A = 3 and B = 4 then det (AB) = 12.
1
10. det A = det A.
⇒ det A =
46 | P a g e
Debre Tabor Univrsity Applied Mathematics II
Prepared by: Dagnaw Tantie
2 7
The minor of 3 is = det = 0 − 7 = −7.
1 0
The cofactor of 3 is (−1) = = −7.
−1 7
The minor of 4 is = det = 0 − 14 = −14.
2 0
The cofactor of 4 is (−1) =− = 14.
−1 2
The minor of 5 is = det = −1 − 4 = −5.
2 1
The cofactor of 4 is (−1) = = −5.
3 5
The minor of 1 is = det = 21 + 5 = 26.
−1 7
The cofactor of 4 is (−1) = = −26.
.................etc
4.3.Adjoint and inverse of a matrix
a) Adjoint of a matrix
∆ , where ∆ = (−1) A .
…
…
adj A = , where is the cofactor of the element .
⋮ ⋮ ⋮ ⋮
…
2 3 1
Example 4.1: Let = −2 4 5 then find adj A.
2 0 7
solution:
4 5
∆ = (−1) A = A = = 28.
0 7
47 | P a g e
Debre Tabor Univrsity Applied Mathematics II
Prepared by: Dagnaw Tantie
−2 5
∆ = (−1) A =− A =− = 24 .
2 7
−2 4
∆ = (−1) A = A = = −8 .
2 0
3 1
∆ = (−1) A =− A =− = −21 .
0 7
2 1
∆ = (−1) A = A = = 12 .
2 7
2 3
∆ = (−1) A =− A =− = −6.
2 0
3 1
∆ = (−1) A = A = = 11 .
4 5
2 1
∆ = (−1) A =− A =− = −12.
−2 5
2 3
∆ = (−1) A = A = = 14.
−2 4
28 −21 11
Therefore, = ∆ = 24 12 −12 .
−8 6 14
−1 3 6
i) = 2 −1 5
−3 2 4
1 0 4
ii) = 2 1 −1
1 0 1
Theorem: If A be a square matrix of order n then A(adj A) = (adj A)A = (det A)I .
( ) ( )
⇒ = , since det A ≠ 0.
( )
⇒ =I .
( )
⇒ A ( ) =A I .
( )
⇒ =A .
( )
Therefore, A =
2 3 1
Example 4.2: Let = −2 4 5 then find the inverse of A.
2 0 7
2 3 1
Solution: we have det A = −2 4 5 = 120. As we did in the above example 4.1,
2 0 7
28 −21 11
adj A = 24 12 −12 .
−8 6 14
28 −21 11
28 −21 11 120 120 120
⎛ 24 12 ⎞
∴A = = 24 12 −12 = 120 120 −12⁄120 .
−8 6 14 −8 6 14
⎝ 120 120 120 ⎠
1 2
Example 4.2: Find if = using determinant.
3 4
49 | P a g e
Debre Tabor Univrsity Applied Mathematics II
Prepared by: Dagnaw Tantie
∆ = (−1) A =− A = − det 2 = −2.
4 −3 4 −2
Therefore , adj A = = .
−2 1 −3 1
1 2
= = −2.
3 4
4 −2 −2 1
∴A = = ⁄ .
−3 1 3 2 −1⁄2
Check that A A = AA = I.
1 0 4
Exercise 4.2: Let = 2 1 −1 then find the inverse of A by using determinant.
1 0 1
Note: Solving a system of equation by this method is known as Matrix inverse method.
2 − 5 = −2
a)
− +3 =4
2 −3 4 1
b) AX=B, where 3 = 0 2 , = 3 .
−1 1 5 2
2 −3 + 3 = −8
c) 3 + 2 + 2 = 10.
− −2 = 1
Solution:
2 −5 −2 3 5
a) AX=B, where = , = . Since = , we get
−1 3 4 1 2
50 | P a g e
Debre Tabor Univrsity Applied Mathematics II
Prepared by: Dagnaw Tantie
3 5 −2 14
= = .
1 2 4 6
2 −3 4 1 1
b) = = 3 0 2 3 = 7 . Hence the solutions are x=1, y=7 and z=12.
−1 1 5 2 12
Theorem: Suppose that A is an × invertible matrix. Then the solution to the system Ax = b
( ) ( ) ( )
is given by, = , = ,…, = ,
3 −2 = 4
a)
2 +5 = 9
2 + 3 + 3 = −8
b) 3 + 2 + 2 = 10.
− −2 = 1
2 −3 3 −8
Solution: we have = 3 2 2 , = and = 10 .
1 −1 −2 1
( )=2 2 2
+3
3 2
+3
3 2
= 2(−2) + 3(−8) + 3(−5) = −43.
−1 −2 1 −2 1 −1
So that
( ) ( ) ( )
= = = = =2.
( ) ( ) ( )
= = = = =3.
51 | P a g e
Debre Tabor Univrsity Applied Mathematics II
Prepared by: Dagnaw Tantie
( ) ( ) ( )
= = = = = −1.
Definition:
1. Suppose that A is a square matrix of order n, ≠ 0 is a vector in and is a scalar
in a field R. Then we say that is an Eigenvector of A with Eigen value if
= . we also say that X is an eigenvecor corresponding to the eigenvalue .
2. The equation det( − ) = 0 is called the characteristic equation of A, while the
polynomial det( − ) is called the characteristic polynomial of A.
Note: The Eigen values of A are the roots of the characteristic polynomial of A.
The Eigen value and the Eigen vector can be computed as follows:
step 1. Solve the characteristic equation det( − )=0 gives the eigenvalues.
step 2. For each Eigen value the corresponding eigenvector is found by substituting
back in to the equation ( − ) = 0.
1 1 6
Example 1: Show that = is an eigenvector of the matrix = corresponding to
2 5 2
the eigenvalue = 3.
1 6
Example 2:Let = . Find the eigenvecors and eigenvalue of A.
5 2
⇔ (1 − )(2 − ) = 0
⇔ − 3 − 28 = 0
⇔ ( − 7)( + 4) = 0
⇔ = 7 or = −4
52 | P a g e
Debre Tabor Univrsity Applied Mathematics II
Prepared by: Dagnaw Tantie
To find the corresponding eigenvector = to the eigenvalue solve
[ − ] =0
If = 7 gives
1 6 1 0
−7 =0
5 2 0 1
1−7 6 0
⇔ =
5 2−7 0
−6 + 6 = 0
⇔
5 −5 = 0
⇔5 −5 =0
⇔ =
Let = = .
Thus the corresponding eigenvectors to the eigenvalue = 7 are nonzero vectors of the
form = .
If = −4 gives
1 6 1 0
+4 =0
5 2 0 1
5 6 0
⇔ =
5 6 0
5 +6 =0
⇔
5 +6 =0
⇔5 +6 =0
⇔ =−
Let = , then = .
Thus the corresponding eigenvectors to the eigenvalue = −4 are nonzero vectors of the
form = .
53 | P a g e
Debre Tabor Univrsity Applied Mathematics II
Prepared by: Dagnaw Tantie
4 0 1
Example 3: Find the eigenvecors and eigenvalue of a matrix = −1 −6 −2 .
5 0 0
solution:
| − |=0
4 0 1 0 0
⇒ −1 −6 −2 − 0 0 =0
5 0 0 0 0
4− 0 1
⇒ −1 −6 − −2 =0
5 0 −
−6 − −2 −1 −2 −1 −6 −
⇒ (4 − ) −0 +1 =0
0 − 5 − 5 0
⇒ +2 − 29 − 30 = 0.
⇒ ( + 1)( − 5)( + 6) = 0
⇒ = −1, = 5, = −6.
For each Eigen value the corresponding eigenvector is found by substituting back in to the
equation ( ( − ) = 0.
If = −1, then
[ + ] =0
54 | P a g e
Debre Tabor Univrsity Applied Mathematics II
Prepared by: Dagnaw Tantie
5 0 1 0
−1 −5 −2 = 0
5 0 1 0
5 + =0
⇒ − −5 −2 =0
5 + =0
5 + =0
⇒
− −5 −2 =0
From the first equation we have, = −5 and substitute back in to the second equation
yields
− −5 − 2(−5 ) = 0
⇒ = .
the form = .
−5
Taking = 5 gives
[ −5 ] = 0
−1 0 1 0
⇒ −1 −11 −2 = 0
5 0 −5 0
− + =0
⇒ − − 11 − 2 = 0
5 −5 =0
− + =0
⇒ , since the first and the third equations are scalar multiple each
− − 11 − 2 = 0
other.
55 | P a g e
Debre Tabor Univrsity Applied Mathematics II
Prepared by: Dagnaw Tantie
So that from the first equation we have, = and substitute back in to the second equation
implies = .
Taking = −6 gives
[ −5 ] = 0
10 0 1 0
⇒ −1 0 −2 = 0
5 0 6 0
10
+ =0
⇒ −2 =0
5 +6 =0
⇒ = =0
2 1
i) =
1 2
0 1 1
ii) = 1 0 1
1 1 0
56 | P a g e
Debre Tabor Univrsity Applied Mathematics II
Prepared by: Dagnaw Tantie
Unit three: limits and continuity
3.1. Definition of limit
i. The intuitively Definition of Limit (Informal Definition)
When we say that “L is the limit of f(x) as x approaches a”, we mean, that f(x) gets close to L as
x gets close to a and we write symbolically by
→ ( )= or ( ) →L as →
Thus, → ( + 2) = 4.
Example 2; If x gets close to 2 then gets close to . In fact the function is not even exist at 2.
but that doesn’t matter because the definition of → says that we consider values of x
that are close to a but not equal to a. The tables below give values of
(correct to four decimal places) for values of that approach 2 (but are not equal to 2).
x<2 f(x) x>2 f(x)
1.9 3.9 2.1 4.1
1.99 3.99 2.01 4.01
1.999 3.999 2.001 4.001
1.9999 3.9999 2.0001 4.0001
On the basis of the values in the tables, we make the guess that
→ =4
Note:
57 | P a g e
Debre Tabor Univrsity Applied Mathematics II
Prepared by: Dagnaw Tantie
1. → ( )= does not imply ( ) = . The function may not even exist at a or may not
equal, some value different than L at a.
2. If he function does not approach a real number L as x approaches a, the limit does not exist;
therefore we write → ( ) does not exist.
There are three ways in which a limit lim → ( ) can fail to exit:
1) f(x) is undefined for some x which is infinitely close but not equal to c.
2) f(x) is infinite for some x which is infinitely close but not equal to c.
3) the standard part of f(x) is different for different numbers x which are infinitely close but
not equal to c.
Examples:
a. lim → √ does not exist because √ is undefined for negative infinitesimal x.(see figure
1.7).
b. lim → does not exist because is infinite for infinitesimal ≠ 0.(see figure 1.7).
does on the other side. For such functions, one sided limit are useful.(see figure 1.7).
Figure 1.7
ii. Precise definition of limit (Formal definition of limit)
58 | P a g e
Debre Tabor Univrsity Applied Mathematics II
Prepared by: Dagnaw Tantie
Definition: Let f be a function defined at each point of some open interval containing a, except
possibly at itself. Then the number L is the limit of f(x) as x approaches a if for every number
> 0 there is a number > 0 such that
→ ( )=
i. → ( )= mean the distance between f(x) and L can be made arbitrarily small by
taking the distance from x to sufficiently small ( but not 0).
ii. If 0 < | − | < , then | ( ) − | < mean
iii. → ( )= mean that for every > 0 (no matter how small is ) we can find
> 0 such that if x lies in the open interval ( − , + ) and ≠ , then f(x) lies in
the open interval ( − , + ).
59 | P a g e
Debre Tabor Univrsity Applied Mathematics II
Prepared by: Dagnaw Tantie
Example 3: Use - definition to show that → (2 − 3) = −1
If = then
60 | P a g e
Debre Tabor Univrsity Applied Mathematics II
Prepared by: Dagnaw Tantie
⟹ | − 1| < =
⟹ | − 1| <
⟹ 2 − 1 < ε
⟹ → (2 − 3) = 1
Solution: For every > 0, we need to find a corresponding number > 0 such that
⇔ |( − 2)| < .
Choose = .
| − 2| < ⇒ 3| − 2| <
⇒ |3 − 6 | <
⇒ |(3 − 2) − 4| <
61 | P a g e
Debre Tabor Univrsity Applied Mathematics II
Prepared by: Dagnaw Tantie
Hence |(3 − 2) − 4| < whenever 0 < | − 2| < .
Therefore, → (3 − 2) = 4.
Solution: Let be a given positive number. We need to find a positive number such that
a) → =
b) → C=
Solution: a) For every number > 0, we need to find a corresponding > 0 such that
∴ → =
Solution: b) For every number > 0, we need to find a corresponding > 0 such that
62 | P a g e
Debre Tabor Univrsity Applied Mathematics II
Prepared by: Dagnaw Tantie
| − |< whenever 0 < | − | < .
Hence, → C=
a) → √ =√
b) → x =4
c) → =
d) → =
Solution a): For every number > 0, we need to find a corresponding > 0 such that
√ √
Now √ − √ = √ −√ √ √
=
√ √
≤ since√ + √ ≥ √ , <
√ √ √ √
⇒ √ −√ ≤ | − |< .
√
⇒| − |≤ √
63 | P a g e
Debre Tabor Univrsity Applied Mathematics II
Prepared by: Dagnaw Tantie
For given > 0 and = √ , if 0 < | − | < = √ then
√ √
√ −√ = √ −√ = ≤ = | − |< √ =
√ √ √ √ √ √ √
∴ → √ =√
Solution b): We must show that for each > 0, there exists > 0 such that
Let = 1.
⇒ | + 2| < 5 … … … … … … . (1)
Solution c): We are asked to show that → = using formal definition of limit.
64 | P a g e
Debre Tabor Univrsity Applied Mathematics II
Prepared by: Dagnaw Tantie
For every number > 0, we must find a corresponding > 0 such that
| |
i.e. − = = = | |
< whenever | − 3| < .
Let = 1. Then
| |
| |
< whenever | − 3| <
⇒ < < .
⇒ =| |
<
| | | |
so that, − = = = | |
≤ < whenever 0 < | − 3| < .
Let = min{1, 6 }.
| |
− = = = | |
≤ | − 3| < (6 ) = .
∴ → =
65 | P a g e
Debre Tabor Univrsity Applied Mathematics II
Prepared by: Dagnaw Tantie
Exercise: Evaluate the following two special trigonometric limits using formal definition.
a) lim → sin =0
b) lim → cos =1
Solution:
2…………………….(∗)
66 | P a g e
Debre Tabor Univrsity Applied Mathematics II
Prepared by: Dagnaw Tantie
0 < | − 0| < ⇒ |sin − 0| < .
∴ lim → sin =0.
b) To show that, lim → cos = 1 from the above figure.
Theorem 1: If L, M, c and k are real numbers and → f(x) = and → g(x) = , then
1) Sum Rule: The limit of the sum of two functions is the sum of their limits.
→ ( f(x) + g(x)) = +
2) Difference Rule: The limit of the difference of two functions is the difference of their
limits.
→ ( f(x) − g(x)) = −
3) Product Rule: The limit of a product of two functions is the product of their limits.
→ ( f(x). g(x)) = .
4) Constant Multiple Rule: The limit of constant times a function is the constant times the
limit of the function.
→ ( k. f(x)) =
5) Quotient Rule: The limit of a quotient of two functions is the quotient of their limits,
provided the limit of the denominator is not zero.
67 | P a g e
Debre Tabor Univrsity Applied Mathematics II
Prepared by: Dagnaw Tantie
( )
→ = ≠0
( )
6) Power Rule: The limit of a rational power of a function is that power of the limit of the
function, provided the latter is a real number.
If r and s are integers with no common factor and ≠ 0, then
⁄ ⁄
→ ( f(x)) =
Examples: Show the following limits using the above theorems.
a) (x + 2x) = 12
→
√
b) =2
→
c) √3x − 1 = √11
→
If ( ) = + + ⋯+ + then
Theorem 3: limits of rational functions can be found by substitution if the limit of the
denominator is not zero.
( ) ( )
If P(x) and Q(x) are polynomials and ( ) ≠ 0, then = .
→ ( ) ( )
f g(x) = f(L).
→
b. √1 − x
→
c. √sin2x
→
∴ √2x − 10 = y = √8 = 2
→ →
Let = siny as → , z→ .
Theorem: Let c be a real number and let ( ) = ( ) for all ≠ in an open interval
containing c. If the limit of g(x) as x approaches c exists, then the limit of f(x) also exist and
f(x) = lim g(x).
→ →
a.
→
√
b.
→
c.
→ √
d.
→ √
( )( )
Solution a): = = (x + x + 1) = 1 + 1 + 1 = 3.
→ → →
√ √
Solution b): lim ( ) = lim ( ) = lim (( )= = .
→ √ → (√ ) → √ ) √ )
√ ( √ ) ( )( )( √ )
Solution c): = lim ( ) = lim = lim =
→ √ → √ √ → →
lim(x) 2 + √x = 4 2 + √4 = 16.
→
69 | P a g e
Debre Tabor Univrsity Applied Mathematics II
Prepared by: Dagnaw Tantie
√ ( )(√ ) ( )(√ )
Solution d): = = = lim √x + 3 +
→ √ √ → → →
2=1+3+2=4.
Exercise:
1. Evaluate the limits
√
a
→
( ⋯ )
b
→
Solution:
√ √ √
1a) = .√ = = = .
→ → → .√ → √
( ⋯ ) ( ⋯
1b) =
→ →
= lim ( + + + ⋯+ )
→
= lim ( 1 + x + 1 + + ⋯+ )
→
⇒ 9(−a) = 9
⇒a =1
a = ±1.
3. = ⇒ =
→ → → →
⇒ = 4
→
70 | P a g e
Debre Tabor Univrsity Applied Mathematics II
Prepared by: Dagnaw Tantie
⇒ = 4 ⇒ a= .
theorem lim → ( ) = 1.
Exercise: Evaluate the following two special trigonometric limits using squeezing theorem.
a) lim → =1
b) lim → =0
Solution a):
71 | P a g e
Debre Tabor Univrsity Applied Mathematics II
Prepared by: Dagnaw Tantie
From the figure we have
Area of ∆ = | || | = (1) =
Area of ∆ = | || | = (1) =
so that, ≤ ≤ .
≤ 1 and ≤ .
∴ lim =1
→
Solution b): Since lim = 0, we cannot apply the quotient theorem directly.
→
= lim → ( )
( )
= lim → ( )
. lim → = 0 × 1 = 0.
72 | P a g e
Debre Tabor Univrsity Applied Mathematics II
Prepared by: Dagnaw Tantie
∴ lim = 0.
→
a) lim
→
b) lim
→
c) lim
→
Solution:
Let = 3 as → 0, → 0.
→
b) lim = lim = lim = = = .
→ → → → →
Exercises:
√
A. Evaluate lim
→ √
√
B. Find the number a and b such that lim = 1.
→
√
C. Evaluate lim , where c is constant.
→
( ) ( )
√
It follows that lim = lim = lim . = lim = lim ( )( )
→ √ → → → →
( )
= lim ( = 1.
→ )
73 | P a g e
Debre Tabor Univrsity Applied Mathematics II
Prepared by: Dagnaw Tantie
Solution (B):Let =√ − ⇒ = − ⇒ = .
As ⟶ 0, ⟶ √− .
√
Thus lim = lim = . lim , exists when b=-1.
→ →√ →√
√
So lim = lim = . lim = . lim ( )(
= . lim ( = =
→ →√ →√ → ) → )
1.
Thus a=3 and b=-1.
As ⟶ 0, ⟶ 1.
√
Thus lim = lim = . lim = . lim ( )(
= . lim ( = .
→ → → → ) → )
1) The limit of the function f(x) as x approaches to a from the left is called left hand limit
and denoted by → f(x) = L.
2) The limit of the function f(x) as x approaches to a from the right is called right hand limit
and denoted by → f(x) = L.
Example 1:
74 | P a g e
Debre Tabor Univrsity Applied Mathematics II
Prepared by: Dagnaw Tantie
Definitions (formal):
a) We say that f(x) has right-hand limit L at a and write
→ f(x) = L.
If for every number > 0 there is a number > 0 such that for all x.
If < < + then | ( ) − | < .
b) We say that f(x) has left-hand limit L at a and write
→ f(x) = L.
If for every number > 0 there is a number > 0 such that for all x
If − < < then | ( ) − | < .
Example 1: Prove that → √x = 0.
Solution: Let > 0 be given. we want to find > 0 such that for all x,
if 0 < < then √x − 0 = √x < .
Now, √x − 0 = √x = √x < implies x < .
So that, if 0 < < then x < .
Choose = .
( ) = .
∴ √x = 0.
→
75 | P a g e
Debre Tabor Univrsity Applied Mathematics II
Prepared by: Dagnaw Tantie
1− <
⇒1− < <1
⇒− < −1<0
⇒ > 1− >0
⇒ <1− <
⇒1− <
⇒ √1 − <
⇒ √1 − <
⇒ √1 − − 0 < .
So that , if 1 − < < 1 then √1 − x − 0 < .
∴ → √1 − x = 0.
Theorem. Let f be a function. Then lim → ( )= if and only if lim → ( )= =
lim → ( ).
3 −2 <1
Example 1: Let ( ) = . Find lim → ( ).
>1
Solution: lim → ( ) = lim → = 1 and lim → ( ) = lim → (3 − 2) = 1.
Therefore, by the above theorem it follows that lim → ( )=1.
1−4 < −2
Example 2: Let ( ) = . Find lim → ( ).
−9 > −2
Solution: lim → ( ) = lim → (1 − 4 ) = −9 and lim → ( ) = lim → (−9) =
−9.
Since, lim → ( ) = lim → ( ) = −9, by the theorem above lim → ( ) = −9.
2 −1 <2
Example 3: Let ( ) = . Find lim → ( ).
3 −2 >2
Infinite Limits
Recall the following definitions of positive infinite limits.
Formally:
i) lim → ( ) = ∞ if for any M there is > 0 such that if x is in ( , + ) then
( ) > .
ii) lim → ( ) = ∞ if for any M there is > 0 such that if x is in ( − , ) then
( ) > .
iii) lim → ( ) = ∞ if both lim → ( ) = ∞ and lim → ( ) = ∞.
Informally:
i) lim → ( ) = ∞ if f(x) can be made arbitrarily large positive by taking x
sufficiently close to a on the right.
ii) lim → ( ) = ∞ if f(x) can be made arbitrarily large positive by taking x
sufficiently close to a on the left.
iii) lim → ( ) = ∞ if f(x) can be made arbitrarily large positive by taking x sufficiently
close to a but not equal to a.
Negative infinite limits are defined analogously as follows.
Formally:
iv) lim → ( ) = −∞ if for any M there is > 0 such that if x is in ( , + ) then
( )< .
v) lim → ( ) = −∞ if for any M there is > 0 such that if x is in ( − , ) then
( )< .
vi) lim → ( ) = −∞ if both lim → ( ) = −∞ and lim → ( ) = −∞.
Informally:
iv) lim → ( ) = −∞ if f(x) can be made arbitrarily large negative by taking x
sufficiently close to a on the right.
v) lim → ( ) = −∞ if f(x) can be made arbitrarily large negative by taking x
sufficiently close to a on the left.
77 | P a g e
Debre Tabor Univrsity Applied Mathematics II
Prepared by: Dagnaw Tantie
vi) lim → ( ) = −∞ if f(x) can be made arbitrarily large negative by taking x
sufficiently close to a but not equal to a.
Example 1: Use the definition to prove lim → = ∞.
2
1
⇔ < whenever 0 < | | < .
1
⇔| |< whenever 0 < | | < .
1
Let > 0 be given and = . If 0 < | − 0| < = then
1
0<| |<
1
⇒0< <
⇒ >
∴ lim → =∞
Thus to check if < 0 and = − be given. If − < − 0 < 0 then < ⇒ > ⇒ < .
∴ lim → = −∞ .
78 | P a g e
Debre Tabor Univrsity Applied Mathematics II
Prepared by: Dagnaw Tantie
If 0 < − 0 < , then > .
We can choose = .
∴ lim → =∞.
b) lim → = lim → ( )( )
=∞
d) lim → = −∞.
e) lim → = ∞.
f) lim → ln = −∞.
Note: If a is a real number then
lim → = ∞.
lim → = −∞.
b. lim →
79 | P a g e
Debre Tabor Univrsity Applied Mathematics II
Prepared by: Dagnaw Tantie
b) We sat that f(x) defined on an open interval (−∞, ) has the limit L as x approaches
negative infinity and we write
lim → ( )= ,
If for every number > 0 there exists a corresponding number N such that for all x,
if < , then | ( ) − | < .
Definition: Informal Definition of Limits Involving Infinity.
1. We say that f(x) has the limit L as x approaches infinity and write
lim → ( )=
if, as x moves increasingly far from the origin in the positive direction, f(x) gets arbitrarily
close to L.
2. We say that f(x) has the limit L as x approaches negative infinity and write
lim → ( )=
if, as x moves increasingly far from the origin in the negative direction, f(x) gets arbitrarily
close to L.
Example: Show that lim → = 0 and lim → = 0.
Solution:
Choose = .
∴ lim → =0
80 | P a g e
Debre Tabor Univrsity Applied Mathematics II
Prepared by: Dagnaw Tantie
if < , then −0 < .
Choose =− .
∴ lim → =0
so).
Step 2 : If k is the highest power of x in the numerator and denominator both, then divide each
term in numerator and denominator by .
81 | P a g e
Debre Tabor Univrsity Applied Mathematics II
Prepared by: Dagnaw Tantie
Step 3 : Use results lim → = 0, when > 0.
Example 3: Determine
1. lim →
2. lim → √
Solution:
1. We divide both the numerator and the denominator by , which is the highest power of x in
the denominator, we obtain
lim → = lim → .
√
lim → = lim →
= lim →
= lim →
( )
= lim → =
√
Exercise:
a. Determine, lim → √ +4 −√
b. Determine, lim →
√ √
c. Determine, lim →
82 | P a g e
Debre Tabor Univrsity Applied Mathematics II
Prepared by: Dagnaw Tantie
f. If lim → ( − − ) = 2, then find the value of a and b.
Solution:
√ √
a. lim → √ +4 −√ = lim → √ +4 −√ .√ √
= lim → √ √
= lim → = lim → = 2.
( ) ( )
b.lim → = lim → = lim → = ∞.
( ) ( )
√ √ √ √
c.lim → = lim → = lim → = .
( ) ( )
√ √
d.lim → √ + +1−√ + 1. √ = lim →
√ √ √
= lim → .
= lim → =
( ) ( )
e. lim → ( − − )=0 ⇒ lim → ( ) = 0.
⇒ lim → ( )=0
( ) ( )
⇒ lim → ( )=0
Since the limit of the given expression is zero, the degree of numerator in the expression must be
less than the degree of denominator. As the denominator is first degree polynomial, the
numerator must be constant, i.e. zero degree polynomial.
⇒ 1− = 0 and + = 0.
⇒ = 1 and = − = −1.
( ) ( )
f. lim → ( − − )=2 ⇒ lim → ( ) = 2.
)
⇒ lim → ( )=2
( ) ( ) )
⇒ lim → ( )=2
83 | P a g e
Debre Tabor Univrsity Applied Mathematics II
Prepared by: Dagnaw Tantie
Since the limit of the given expression is finite non-zero number , the numerator and the
denominator are of the same degree.
⇒ 1− =0 ⇒ =1
( ) ( ) ) ( )
Now lim → ( )=2 ⇒ lim → ( ) = 2.
( )
1
(− − − − )
⇒ lim ( )=2
→ 1
(1 + )
⇒ − − =2 ⇒ = − − 2 = 1 − 2 = −3 .
ii. lim →
iii. lim → ln
Horizontal and Vertical Asymptotes
Definition:
1. The line = is a horizontal asymptote of the graph of a function = ( ) if either
lim →∞ ( )= or lim ( )= .
→ ∞
±∞ .
asymptote of f. The limit, "as → −∞", is also 2. So = 2 is the only horizontal asymptote f.
The graph of f and = 2 are given in figure 7. A function may or may not cross its asymptote.
86 | P a g e
Debre Tabor Univrsity Applied Mathematics II
Prepared by: Dagnaw Tantie
Example 2: Find the vertical asymptotes of ( ) = and ( ) = .
( )( )
Solution: ( ) = = . So the only values which makes the denominator zero are =0
( )
( )
Similarly, ( ) = = . So the only candidates to be vertical asymptotes are = 0 and =1
( )
( −3) ( −3)
lim → ( ) = lim → = lim → = − ∞.So = 1 is a vertical asymptote of g.
( −1) ( −1)
( −3) ( −3)
lim → ( ) = lim → = lim → ( −1) = 3 ≠ −∞.So = 0 is not vertical asymptote of g.
( −1)
If the limit of f(x) as " → ∞" or " → −∞" is constant K, then the graph of f gets close to the
horizontal line = , and we said that = was the horizontal line of f. Some functions, however,
approaches other lines which are not horizontal.
87 | P a g e
Debre Tabor Univrsity Applied Mathematics II
Prepared by: Dagnaw Tantie
88 | P a g e
Debre Tabor Univrsity Applied Mathematics II
Prepared by: Dagnaw Tantie
1.7. Continuity
Continuity Test
A function f is said to be continuous at a point a if
i. f(a) is defined; (a lies in the domain of ƒ )
ii. lim → ( ) exists, and (ƒ has a limit as → )
iii. lim → ( ) = ( ). (the limit equals the function value)
Note: A function f(x) is called everywhere continuous (or often, just "continuous") if it is
continuous at x = a for all real numbers a.
Examples: Determine the number at which f is continuous for the following functions.
a) ( )=
≠3
b) ( )=
5 =3
≠3
c) ( )=
2 =3
d) ( )=
Solution: a)Since f(x) is defined for all ∈ ∖ {−1,1}, f is continuous for all ∈ ∖ {−1,1}.
89 | P a g e
Debre Tabor Univrsity Applied Mathematics II
Prepared by: Dagnaw Tantie
b)The function is defined for all real numbers and lim → ( ) = ( ), ∀ ∈ ∖ {3}. So f is
continuous for all ∈ ∖ {3}.
( )( )
But lim → ( ) = lim → = lim → + 2 = 5 and (3) = 5. Hence f is continuous
90 | P a g e
Debre Tabor Univrsity Applied Mathematics II
Prepared by: Dagnaw Tantie
Theorem: If the functions ƒ and g are continuous at x = a, then ƒ + g, ƒ − g, ƒ . g ,
k. f (k is constant) and ƒ/g (g(a) ≠ 0) are continuous at x = a.
Types of discontinuity
The kind of discontinuity illustrated in parts (a) and (c) is called removable because we could
remove the discontinuity by redefining f at just the single number 2. [The function g(x)=x+1 is
continuous.] The discontinuity in part (b) is called an infinite discontinuity. The discontinuities
in part (d) are called jump discontinuities because the function “jumps” from one value to
another.
+1 <0
Example 1: Show that the function f defined by ( ) = is continuous from
2 ≥0
the right at 0 but not continuous from the left at 0.
Solution: lim → ( ) = lim → 2 = 0 and (0) = 2(0) = 0. Since lim → ( )=
0= (0), f is continuous from the right at 0.
lim → ( ) = lim → + 1 = 1 and (0) = 2(0) = 0. Since lim → ( ) = 1 ≠ (0) = 0,
f is not continuous from the left at 0.
≤1
Example2 : Let ( ) = . Show that f is continuous at 1.
>1
Solution: lim → ( ) = lim → = 1 and lim → ( ) = lim → = 1.
Thus f is continuous from the left and from the right at 1.
Therefore f is continuous at 1.
91 | P a g e
Debre Tabor Univrsity Applied Mathematics II
Prepared by: Dagnaw Tantie
Theorem (Continuity and Limits)
Suppose that lim → ( ) = L and that f is a function continuous at L. Then
lim → ( ( )) = ( lim → ( )).
Theorem:
i. Any polynomial function is continuous everywhere; that is, it is continuous on =
(−∞, ∞).
ii. Any rational function is continuous whenever it is defined; that is, it is continuous on its
domain.
Continuity on Intervals
Definition (Continuity on Open Intervals)
If a and b are real numbers with a < b, and if f is a function, we say that f is continuous on (a, b)
when f is continuous at each point of (a, b).
Definition (Continuity on a Closed Interval)
The function f is continuous on [a, b] if f is continuous on (a, b), f is continuous from right at a,
and f is continuous from the left at b.
Example: Sow that the function ( ) = √4 − is continuous at every point of its domain,
[−2, 2].
92 | P a g e
Debre Tabor Univrsity Applied Mathematics II
Prepared by: Dagnaw Tantie
iii. Continuous function is one that is continuous at every point of its domain.
93 | P a g e
Debre Tabor Univrsity Applied Mathematics II
Prepared by: Dagnaw Tantie
Unit Four
= lim → ,
( )
( )( )
= lim → ,
( )( )
= lim → ,
94 | P a g e
Debre Tabor Univrsity Applied Mathematics II
Prepared by: Dagnaw Tantie
= lim → ( )(
,
)
= lim → ( )(
,
)
=( )(
=( .
) )
Definitions:
1) The slope of the curve = ( ) at the point ( , ( )) is the number
( ) ( )
= lim → , (provided the limit exists).
2) The tangent line to = ( ) at ( , ( )) is the line through ( , ( )) whose slope is
equal to ′( ), the derivative of f at .
3) The normal tangent line to = ( ) at ( , ( )) is the line through ( , ( )) whose
slope is equal to ′( ).
Note: We can write the equation of the tangent line to the curve = ( ) at the point ( , ( )):
− ( ) = ( )( − ).
Thus, the geometric interpretation of a derivative [as defined by either (2) or (3)] is as shown in
Figure 1.
95 | P a g e
Debre Tabor Univrsity Applied Mathematics II
Prepared by: Dagnaw Tantie
√ √
= lim →
√ √ √ √
= lim → . .
√ √
= lim → ..
(√ √ )
= lim → ..
(√ √ )
= lim → (√ ..
√ )
=( = .
√ √ ) √
1
The derivative is, ( )=
2√
.
Now all that we need is the function value and derivative (for the slope) at x 3 .
1
(3) = √3 and = (3) = .
2 √3
1
The tangent line is then, = √3 + ( − 3).
2 √3
Exercise: Find the equation of the tangent line to ( ) = 4 − 8 √ at x 16 .
Solution: We know that the equation of a tangent line is given by, y f af ax a
So, we will need the derivative of the function (don’t forget to get rid of the radical).
( )= 4 −8 ⇒ ( )= 4−4 =4− ..
√
Again, notice that we eliminated the negative exponent in the derivative solely for the sake of the
evaluation. All we need to do then is evaluate the function and the derivative at the point in question,
x 16 .
Note that the converse of this theorem is not true. That is, every continuous function may not be
differentiable.
( ) ( )
′ (0) = lim → does not exist implies f is not differentiable at x=0.
Exercise:
( ) ( )
Solution: a) we have lim → = lim → = lim → ( + 1) = 2 and
( ) ( )
lim → = lim → = lim → ( + 1) = 2.
( ) ( )
So, lim → = 2 = ′(1). Thus f is differentiable at x=1 and ′(1) = 2.
( ) ( ) ( ) ( ) ( ) ( )
Solution d) lim → = 1 and lim → = −1. So lim → does not exist
1) ( ) = 0
2) ( ) = 1
3) ( ) =
97 | P a g e
Debre Tabor Univrsity Applied Mathematics II
Prepared by: Dagnaw Tantie
4) ( ( ) ± ( )) = ′( ) ± ′( )
5) ( ( ). ( )) = ( ). ( ) + ( ). ( )
( ) ( ). ( ) ( ). ( )
6) ( ) =
( ) ( ( ))
( )
7) ( ) =(
( ) ( ))
a) =
b) =
c) ( ) = ln 2
d) ( ) = √ + 9√ −
√
e) ( ) = (3 − 7)(4 + 2 − 5)
f) ( )=
Theorem (The Chain Rule): If g is differentiable at x and f is differentiable at g(x), then the
composite function F = f ∘ g defined by F(x) = f(g(x)) is differentiable at x and ′ is given by the
product
F'(x) = f '(g(x)) · g'(x)
Note: In Leibniz notation, if y = f (u) and u = g(x) are both differentiable functions, then
= . .
Example: Find the derivative of the following functions using chain rule.
a) =( − 1) .
b) ( )= .
c) ( ) = (2 − 4) . (2 + 2 + 1) .
d) ( ) = sin(cos( )).
e) ( ) = 2sin(3cos( 4 )).
f) = √sin .
98 | P a g e
Debre Tabor Univrsity Applied Mathematics II
Prepared by: Dagnaw Tantie
Solutions:
a) =( )( ) = ( ( )), where = ( )= − 1 and = ( )= .
= . = 100 .3 .
⇒ = 100( − 1) .3 = 300( − 1) .
b) ( )=8 . ( ).
( ). ( ) ( ). ( )
=8 .
( )
( ). ( ).
=8 .
( )
( )
=8 = .
( ) ( )
c) ( ) = (2 + 2 + 1) . (2 − 4) + (2 − 4) . (2 + 2 + 1) .
= (2 + 2 + 1) . 7(2 − 4) . (2 − 4) + (2 − 4) . 5(2 + 2 + 1) . (8 + 2)
= (2 + 2 + 1) . 7(2 − 4) . 2 + (2 − 4) . 5(2 + 2 + 1) . (8 + 2)
= 14(2 + 2 + 1) . (2 − 4) + 5(2 − 4) . (2 + 2 + 1) . (8 + 2) .
Exercise:
1. Suppose that ( ) = ( ( )) and (3) = 6, ′(3) = 4, ′(3) = 2, and ′(6) = 7.Then find
′(3).
2. Let (3) = 2 and ′(3) = 3. Then find ′( 3 ) .
a) ( ) = ( )
b) ( ) = [ ( − 24)]
c) ( ) = (6 − )
d) ( ) = ( )
( )
e) ( ) = ( )
99 | P a g e
Debre Tabor Univrsity Applied Mathematics II
Prepared by: Dagnaw Tantie
1) (sin x) = cos x
2) (cos x) = −sin x
3) (tan x) = sec x
4) (sec x) = sec x . tan x
5) (csc x) = −csc x . cot x
6) (cot x) = −csc x
The chain rule will help us to determine the derivative of logarithms and exponential functions
. We will also use it to answer some applied questions and to find the slope of graph given by
parametric equations.
i. (e ) = e .
ii. (e ( ) ) = e ( ) . f′(x).
v. (a ) = ln . a .
vi. (a ( ) ) = ln . a ( ) . f′(x).
( ) ( ) ( )
ix. ( ) = ( ) ( ′( ) ( ( ( ))) + ( ). .
( )
.
Proof i: (e ) = lim → = lim → = e . lim → = e .1 = e .
( )
iii: Let ( ) = ln . Then = . Taking both side derivative with respect to x,
( ) ( ) ( )=
1= ⇒ ( ) = .
Exercises: Find ′( ) if
i. ( )= .
ii. ( )= .
iii. ( )= ( )
iv. ( )=2 .
v. ( )= .
vi. ( ) = ( + 2) .
Solutions:
i. ( )=( ) + =3 . + .2 = 3 +2 .
( ) . .( ) . .
ii. ( )= = = .
iii. ( )= = = = 2 . Simply = .
iv. ′( ) = (2 ) = ln2. 2 .
v. ( )= ( ) =3 .
( )= ( + 2) . ( ) . ( ) (2 . ln( + 2))
vi. = ( )= .
( )
= ( + 2) (2 ln( + 2) + 2 )
= ( + 2) (2 ln( + 2) + )
101 | P a g e
Debre Tabor Univrsity Applied Mathematics II
Prepared by: Dagnaw Tantie
( )′ .The function ′′ is called the second derivative of ƒ because it is the derivative of the first
derivative. Notationally,
′
f ′′ (x) = = = = y ′′ = D (f)(x) = D f(x) .
The nth derivative of y with respect to x for any positive integer n, denoted as
y( )
= y = = D y.
A convenient notation is
( )( )= , which read as " the nth derivative of f with respect to x."
and
102 | P a g e
Debre Tabor Univrsity Applied Mathematics II
Prepared by: Dagnaw Tantie
equation. when this functions are differentiable, it is possible to find their derivatives without
solving the equation for y. The method is called implicit differentiation.
Example: Find
√ √
a. + = 1. Find the equation of the tangent line to the curve at the point , .
b. =3 .
c. + = + 1.
d. sin( ) = .
e. cos( + ) = .
Solution: a) ( + )= 1.
⇒ 2 +2 . =0 ⇒ 2 . = −2 ⇒ =− .
b) ( )= (3 ) ⇒ ( )+ ( )=3 ( ( )+ (3 )).
⇒ (3 )+ (3 )=3 +3 .
⇒ (3 )−3 =3 −3 ).
⇒ (3 −3 ) =3 −3 .
⇒ =( = .
)
d) sin( )= ⇒ cos( ). ( )=1
⇒ cos( ). ( . + . )=1
⇒ cos( ). ( + . )=1
⇒ ycos( ) + xcos( ). =1
⇒ xcos( ). =1− cos( )
( )
⇒ = ( )
103 | P a g e
Debre Tabor Univrsity Applied Mathematics II
Prepared by: Dagnaw Tantie
i. cos =
ii. =
Theorem: Implicit Differentiation If ( , ) = 0 then
∕
=− .
∕
104 | P a g e
Debre Tabor Univrsity Applied Mathematics II
Prepared by: Dagnaw Tantie
Trigonometric functions and derivatives of Trigonometric integrals
1. [ ]= .
2. [ ]=− .
3. [ ]= .
4. [ ]= .
| |
5. [ ]=− .
6. [ ]=− .
| |
Hyperbolic Functions
INTRODUCTION
In this section, we introduce a new group of mathematical functions, based on the functions
and whose properties resemble, very closely, those of the standard trigonometric functions.
But, whereas trigonometric functions can be related to the geometry of a circle (and are
sometimes called the “circular functions”), it can be shown that the new group of functions are
related to the geometry of a hyperbola. Because of this, they are called “hyperbolic functions”.
I. Definition of Hyperbolic Functions
There are six hyperbolic functions and they are defined as follows.
3. Hyperbolic cosine: The hyperbolic cosine of a number, x, is denoted by cosh x
Sãsä_R- TVé/ìsUX W (ãqY[Hñ .AZ\ äW^å ç.][ é2ð O[íOò _'aê }ïsW(îsY: òâ.b _íé 2îsce 8èdf ±� R=è giíîs _ãh L� u_ _j[_? TksM â g 8ë îsl m_ã R $� uc e__ _?T klM ò .Zé â _ ã � _ __5 _ ë îs ì ò.è ò _ ã � _ ___ Gãlè v ä.ë ò q2 _ãä _� _í O__% î_ Gq2 _äå.ì sè_ é.ä.wè îsN)é vâãs [ä.î _y ò .$éé 2_) îs.ä è_q2 íOìsä )vå ä0M å.äâ .ì.ãs ä.= é q2. ääq 2åNä .îâ.ã sé2 ä.îsíow åä. è.N) ãsìsé 2éÅðOä _ãlvë ìsq2 é2ä _æ .íOë % î í Oq2 _îä MHå.N )è_ä .ãsèWí OðíO ä_0 ê â. _)ãs ä.ä7 ò..ä é2q 2îsä. î.è_ íOìsé 2äîs å.ä.x èâ.ãs ±� äòæ é 2è _ãé =� _q 2__ ä)_ _å.ë â.îs ãsìä å.ä. è.â å.íOò )â. ä.ì²ì sé2æ .ë íOî_ _{z^# _ &ã sä.íOî% 2q äå. è ä.ã qñHAEä) å çEé2 ðíOò .ê}ïs îòâ. íé2îs è_)N íOð â.ãs ä.î² ãlëv2 qä.â ãsä. ê}é 2ðO é$) _íî_* ìs é2æ .ë íî sè_ ë îsì. å ë î _* 2ä.è -|
III.
Inverse Hyperbolic Functions
i. =
ii. =
iii. =
iv. =
| |
v. =
| |
vi. =
Proof i) y = sinh ⟺ = sinhy =
⟺ = ( )
⟺ =
⟺ 2e = e −1
⟺e − 2e + 1 = 0
106 | P a g e
Debre Tabor Univrsity Applied Mathematics II
Prepared by: Dagnaw Tantie
√
⟺e = = x + √x + 1
⟺ = ln(x + √x + 1 )
Thus y = sinh = ln x + √x + 1
⟺ = ( )
⟺ =
⟺ e + =e −1
⟺ e (x − 1) = −x − 1
( )
⟺e = −( )
( )
⟺ 2y = ln (− ( )
)
( )
⟺ y = ln (− ( )
)
( )
Thus y = tanh = ln (− ( )
)
107 | P a g e
Debre Tabor Univrsity Applied Mathematics II
Prepared by: Dagnaw Tantie
Definition: We say that f(x) has
( , ) containing c
Note:
i. An absolute maximum or absolute minimum of f is called an absolute extremum of f.
ii. A local maximum or minimum of f is called a local extremum of f.
Example: Identify the absolute extrema and relative extrema for the following function.
Function rule Domain Absolute a extrema on D Relative extrema on D
D
a) ( )= (−∞, ∞) No absolute maximum. No relative maximum.
Absolute maximum of 0, relative maximum of 0,
at =0 at =0
b) ( )= [0, 3] Absolute maximum of 9, Has no relative
at = 3. extrema.
Absolute minimum of 0, at
108 | P a g e
Debre Tabor Univrsity Applied Mathematics II
Prepared by: Dagnaw Tantie
= 0.
c) ( )= (0, 3] Absolute maximum of 9, Has no relative
at = 3. extrema.
No absolute minimum.
d) ( )= (0, 3) No absolute extrema. Has no relative
extrema
e) ( )= [−2, 2] Absolute maximum of 8 at no relative extrema.
= 2 and an absolute
minimum of -8 at x = -2.
Critical Points
Definition: We say that = is a critical point of the function ( ) if ( ) exists and if either
of the following are true; ′( ) = 0 Or ′( ) doesn't exist
Example: Determine all the critical points for the functions,
a) ( ) = 6 + 33 − 30 + 100 .
b) ( ) = √t (2t − 1)
c) ( ) = ln(3 ) + 6.
Solution a): We first need the derivative of the function in order to find the critical points and so
let’s get that.
′( ) = 30 + 132 − 90 .
= 6 ( + 22 − 15) .
= 6 (5 − 3)( + 5) .
Now, our derivative is a polynomial and so will exist everywhere. Therefore the only critical
points will be those values of x which make the derivative zero. So, we must solve.
6 (5 − 3)( + 5) = 0
Form this we can see that there are three critical points. They are,
= 0, = , = −5.
Note: Polynomials are usually fairly simple function to find critical points for provided the
degree doesn’t get so large that we have trouble finding the roots of the derivative.
Solution b): To find the derivative it’s probably easiest to do a little simplification before we
actually differentiate.
( ) = t (2t − 1) = (2t − t ) .
Now differentiating this gives
( )
= t − t = t − .
109 | P a g e
Debre Tabor Univrsity Applied Mathematics II
Prepared by: Dagnaw Tantie
We will need to be careful with this problem. The derivative of this function doesn’t exist at t=0
and so that will be one critical point. However, we will also need to determine where the
derivative is zero (provided it is of course…).
To help with this it’s usually best to combine this into a single rational expression. So,
getting a common denominator and combining gives us,
( )
= .
Notice that we still have t=0 as a critical point. It’s also become much easier to quickly
determine where the derivative will be zero. Recall that a rational expression will only be zero if
its numerator is zero (and provided the denominator isn’t also zero at that point of course).
The numerator will be zero if = and so there are two critical points for this function, = 0
and = .
Solution c): Before getting the derivative let’s notice that since we can’t take the log of a
negative
number or zero we will only be able to look at > 0.
The derivative is then,
′( ) = (2 ln(3 ) + 1).
Now, this derivative will not exist if x is a negative number or if x=0, but then again neither will
the function and so these are not critical points. If x>0 the function will exist and so the only
thing we need to worry about is where the derivative is zero. First note that despite appearances
the derivative will not be zero for x=0. For x=0 the derivative doesn’t exist because of the
natural logarithm and so the derivative can’t be zero there!
So, the derivative will only be zero if,
(2 ln(3 ) + 1) = 0 ⇒ = .
√
Note:
The relative extrema of a function, if any occur at critical points.
110 | P a g e
Debre Tabor Univrsity Applied Mathematics II
Prepared by: Dagnaw Tantie
Fermat’s Theorem doesn’t say that a critical point will be a relative extrema. For instance , the
function ( ) = has no relative extrema. But clearly x 0 is a critical point.
An absolute extrema may or may not be a critical point.
Theorem (Extreme Value Theorem)
If f is continuous on a closed interval [a, b], then it has both a minimum and a maximum point.
That is, there are real numbers c and d in [a, b] so that for every x in [a, b], f(x) ≤ f(c) and f(x) ≥
f(d).
A Procedure for Finding the Absolute Extrema of a Continuous Function f on a Finite
Closed Interval [a, b].
Step 1. Find the critical points of in (a, b).
Step 2. Evaluate f at all the critical points and at the endpoints a and b.
Step 3. The largest of the values in Step 2 is the absolute maximum value of f on [a, b] and the
smallest value is the absolute minimum.
Example: Determine the absolute extrema values for the following functions.
a) ( ) = 2 + 3 − 12 + 4 on 4, 2
b) ( ) = 3 ( + 4) on [−5, −1]
c) ( )= − + 4 − 3 on , 1
d) ( ) = 7 + | | on , 4
e) ( )= − on 4, 2
Solution a): The function ( ) polynomial and hence everywhere and in particular is then
continuous on the given interval.
Now, we need to get the deri+ative so that we can find the critical points of the function.
′( ) = 6 + 6 − 12 = 6( + 2)( − 1) .
Thus, we’ll have two critical points, x 2 and y 1.
Now we evaluate the function at the critical points and the end points of the interval.
(−4) = −28
(−2) = 24
(1) = −23
111 | P a g e
Debre Tabor Univrsity Applied Mathematics II
Prepared by: Dagnaw Tantie
(2) = 8
Absolute extrema are the largest and smallest the function will ever be and these four points
represent the only places in the interval where the absolute extrema can occur. So, from this list
we see that the absolute maximum of g(x) is 24 and it occurs at x = -2 (a critical point) and the
absolute minimum of g(x) is -28 which occurs at x = -4 (an endpoint).
Solution b): Again, as with all the other examples here, this function is continuous on the given
interval and so we know that this can be done.
First we’ll need the derivative and make sure you can do the simplification that we did here to
make the work for finding the critical points easier.
( ) = 3( + 4) + 3 . . ( + 4) = 3( + 4) +
( )
( )
= =
( ) ( )
So, if we had ignored or forgotten about the critical point where the derivative doesn’t exist ( x 4 ) we
would not have gotten the correct answer.
Theorem (Rolle's Theorem): Let f be continuous on the closed interval [a, b] and differentiable
on the open interval (a, b). If f (a) = f(b) then there is at least one point c in the interval (a, b)
112 | P a g e
Debre Tabor Univrsity Applied Mathematics II
Prepared by: Dagnaw Tantie
such that ′( ) = 0.
Example 1: Verify Rolle's Theorem and find the number that satisfying the theorem:
a. ( )= − 2 on [0,2]
b. ( )= on [−2,2]
′( ) = 0.
Theorem (Mean-Value Theorem): Let f be continuous on the closed interval [a, b] and
differentiable on the open interval (a, b). Then there is at least one point c
in (a, b) such that
113 | P a g e
Debre Tabor Univrsity Applied Mathematics II
Prepared by: Dagnaw Tantie
( ) ( )
′( ) = or ( ) − ( ) = ′( )( − )
Example: Determine all the numbers c which satisfy the conclusions of the Mean Value
Theorem for the following functions.
a. ( ) = + 2 − on [-1, 2]
b. ( )= − on [3, 4]
Solution a: Since f (x) is a polynomial it is both continuous and differentiable (i.e. the derivative
exists) on the interval given.
First let’s find the derivative.
′( ) = 3 + 4 −1.
Now, to find the numbers that satisfy the conclusions of the Mean Value Theorem all we need to
do is plug this into the formula given by the Mean Value Theorem.
( ) ( )
( )= ( )
= =4
⇒3 + 4 −1=4
⇒3 + 4 −5=0
±√ ±√
= = .
√ √
= = 0.7863, Or = = −2.1196.
Notice that only one of these is actually in the interval given in the problem. That means that we
will exclude the second one (since it isn’t in the interval). The number that we’re after in this
problem is, = 0.7863
114 | P a g e
Debre Tabor Univrsity Applied Mathematics II
Prepared by: Dagnaw Tantie
Exercise:
1. Let ( ) = . Find a value ∈ (−1, 2) so that ′( ) equals the slope between the
endpoints of f(x) on [−1, 2].
2. Verify that f(x) = x/(x + 2) satisfies the hypotheses of the Mean Value Theorem on the
interval [1, 4] and then find all of the values, c, that satisfy the conclusion of the theorem.
3. Verify that f(x) = 3x/(x + 7) satisfies the hypotheses of the Mean Value Theorem on the
interval [−2, 6] and then find all of the values, c, that satisfy the conclusion of the
theorem.
4. Let f(x) = tan x. Show that ( ) = (2 ) = 0 but there is no number c ∈ ( , 2 ) such
that ′( ) = 0. Why does this not contradict Rolle’s Theorem?
5. Let ( ) = ( − 3) . Show that there is no value c (1, 4) such that ′( ) = ( (4) −
(1))/(4 − 1). Why is this not a contradiction of the Mean Value Theorem?
Facts
Definition: Let ƒ be a function defined on an interval I and let and be any two points in I.
115 | P a g e
Debre Tabor Univrsity Applied Mathematics II
Prepared by: Dagnaw Tantie
Example: Find the intervals for which ( ) = − 4 + 3 is increasing and the intervals on
which it is decreasing.
Solution: The derivative of f is
( ) = 2 − 4.
It follows that,
′( ) > 0 if >0
( ) < 0 if <0
Since f is continuous everywhere, it follows that
f is increasing on [2, +∞)
f is decreasing on (−∞, 2]
Exercise: Find the interval on which f is increasing and the intervals on which it is decreasing for
the following functions.
a. ( )=
b. ( )=
c. ( )=3 +4 − 12 +2
Theorem (First Derivative Test): Suppose that f is continuous at a critical point x = c then,
a. If f x0 to the left of x c and f x0 to the right of x c then x c is a relative
maximum.
b. If f x0 to the left of x c and f x0 to the right of x c then x c is a relative
minimum.
c. If f xis the same sign on both sides of x c then x c is neither a relative maximum nor a
relative minimum.
Alternative theorem for, first derivative test
Theorem(FDT): Suppose that c is a critical point of a continuous function ƒ, and that ƒ is
differentiable at every point in some interval containing c except possibly at c itself.
Moving across c from left to right,
i. if ' changes from negative to positive at c, then ƒ has a local minimum at c;
ii. if ' changes from positive to negative at c, then ƒ has a local maximum at c;
iii. if ' does not change sign at c (that is, ' is positive on both sides of c or negative on
both sides), then ƒ has no local extremum at c.
117 | P a g e
Debre Tabor Univrsity Applied Mathematics II
Prepared by: Dagnaw Tantie
a) all critical points of f.
b) the interval where the function f is increasing and decreasing.
c) the relative maximum and minimum of f.
Solution (a): The function f is defined an continuous for all real values of x, and its derivative is
′ ( )
( )=5 − 10 =5 ( − 2) =
′ ′( ) = 0 if
Since ( ) does not exist if = 0 and since = 2, there are critical points at
= 0 and = 2.
Solution (b): Here is a number line with the critical points graphed and test points.
′(−1) = 15 (1) = −5 ′(8) = 15
( )>0 ( )<0 ( )>0
Increasing 0 Decreasing 2 Increasing
Thus f is increasing on the interval (−∞, 0] and [2, ∞], and f is decreasing on the interval [0,2].
Solution (c): Since ′( ) > 0 to the left at = 0 and ( ) < 0 to the right of = 0 then =0
is a relative maximum. And since ( ) < 0 to the left at = 2 and ( ) > 0 to the right of
= 2 then = 2 is a relative minimum point.
OR
To apply the first derivative test, we examine the sign of ′( ) on intervals extending to the left
and to the right of the critical points. Since the sign of derivative change from positive to
negative at x=0 , there is a relative maximum there, and since it changes from negative to
positive ax x=2, there is a relative minimum there.
Exercise: Locate the relative extrema for the following functions using first derivative test, if any
a) ( )= −3 +3 −1
b) ( )= √ − 4.
ii. Second Derivative Test for Local Extrema
Theorem (Second Derivative Test): Suppose that f is twice differentiable at the point c and
′′( ) is continuous on an open interval that contains = .
1) If ( ) = 0 and ( ) < 0 then ƒ has a local maximum at = .
2) If ( ) = 0 and ( ) > 0 then ƒ has a local minimum at = .
118 | P a g e
Debre Tabor Univrsity Applied Mathematics II
Prepared by: Dagnaw Tantie
3) If ( ) = 0 and ( ) = 0 then the list in conclusive: that is, f may have a relative
maximum, a relative minimum, or neither at c.
Example: Find the relative maxima and minima of ( )= −2 .
Solution: First we find the derivative of f to find critical points
( )=4 −4 =4 ( − 1) = 4 ( − 1)( + 1)
( ) = 12 −4.
Solving ′( ) yields the critical points = 0, = 1 and = −1.
Evaluating f at these points yields
(0) = −4 < 0
(1) = 8 > 0
(−1) = 8 > 0
So there is a relative maximum at = 0 and a relative maxima at = 1 and = −1.
Exercise: Find the relative maxima and minima of
a)
b)
Theorem: Suppose that f is continuous and has exactly one relative extremum on an interval, say
at = .
a) If f has a relative minimum at = , then ( ) is the absolute minimum of f on the
interval.
b) If f has a relative maximum at = , then ( ) is the absolute maximum of f the interval.
First Derivative Test for Absolute Extreme Values
Suppose that c is a critical number of a continuous function f defined on an interval.
a) If ′( ) > 0 for all < and ′( ) < 0 for all > , then ( ) is the absolute
maximum value of f.
′
b) If ′( ) < 0 for all < and ( ) > 0 for all > , then ( ) is the absolute
minimum value of f.
For instance, ( ) = has absolute minimum value at = 0, because ′( ) < 0 for all <0
′
and ( ) > 0 for all > 0.
3.3 Concavity and inflection points
119 | P a g e
Debre Tabor Univrsity Applied Mathematics II
Prepared by: Dagnaw Tantie
Definition: If the graph of f lies above all of its tangent on an interval I, then it is called concave
upward on I. If the graph of f lies below all of its tangent on I, it is called concave down ward
on I.
Alternatively it can be defined as
Definition: The graph of a differentiable function f(x) is
a. concave up on an open interval I if ′ is increasing on I.
b. concave down on an open interval I if ′ is decreasing on I.
Definition: A point x=c is called an inflection point if the function is continuous at the point
and the concavity of the graph changes at that point.
The Second Derivative Test for Concavity
Theorem: Let f be twice differentiable on an open interval.
a) If ′′( ) > 0 for all x in I, then the graph of f is concave up ward on I.
b) If ′′( ) < 0 for all x in I, then the graph of f is concave up ward on I.
Example: Find the interval of concave up and concave down, and locate all inflection points for
the following functions.
a) ( )= −3 +1
b) ( )=3 −5 +3
Solution (a): The first and second derivatives of f are
( )=3 − 6 = 3 ( − 2)
′′( ) = 6 − 6 = 6( − 1)
120 | P a g e
Debre Tabor Univrsity Applied Mathematics II
Prepared by: Dagnaw Tantie
Interval 6( − 1) Sign of ( ) Conclusion
<1 (−) − f is concave down on
(−∞, 1)
>1 (+) + f is concave up on (1, +∞)
The second table shows that there is an inflection point at x = 1, since f changes from concave
down to concave up at that point. The inflection point is (1, (1)) = (1, −1).
Solution (b): The first and second derivatives of g are
( ) = 15 − 15 = 15 ( − 1) = 15 ( − 1)( + 1)
( )
= 60 − 30 = 60 − = 60 − + .
√ √
1 (−)(−)(+) + g is concave up on − ,0
− < <0 √
√2
1 (+)(+)(−) − g is concave down on 0,
0< < √
√2
1 (+)(+)(+) + g is concave up on ,∞
> √
√2
Thus =− , = 0, = are all inflection points, since f changes, from concave down to
√ √
concave up, from concave up to concave down, from concave down to concave up at these points
Solution:
never zero for whatever values of x, f has no critical number and hence no local extrema.
2. Since ( )= > 0 ∀ ≠ −1. We conclude that f is increasing in its domain.
( )
3. To determine the interval for which f is concave up and concave down, take the second
derivative of f.
( )
( )=− = −(
( ) )
Interval ( + 1) 4 Conclusion
( )=−
( + 1)
< −1 − + f is concave up (−∞, −1)
> −1 + − f is concave down (−1, ∞)
Since f is not continuous ax = −1, it has no inflection point
122 | P a g e
Debre Tabor Univrsity Applied Mathematics II
Prepared by: Dagnaw Tantie
8
x 1
g( x) =
x+1
6
y=1
q( y) = 1
4
15 10 5 5 10 15
Related Rates
Related rates means related rates of change, and since rates of changes are derivatives, related
rates really means related derivatives.
The procedure to solve a related rates problem:
1. Write down the rate which is given.
2. Write down the rate which is unknown.
3. Write down your notation and draw a diagram.
4. Find a formula connecting the quantities you listed in your Notation.
5. Implicitly differentiate the formula to get rates of change involved. If you end up with more
than one unknown rate of change, you might have to eliminate a variable using geometry (as
mentioned in the previous step).
6. Solve for the Unknown Rate.
7. Substitute values to determine the Unknown Rate.
8. Write a concluding sentence.
123 | P a g e
Debre Tabor Univrsity Applied Mathematics II
Prepared by: Dagnaw Tantie
Example: Two cars start moving from the same point. One travels south at 25 mph, and the other
travels west at 15mph. As what rate is the distance between the cars increasing 2 hours later?
1. Given: The rate of change of the distance (or velocity) of a car travelling south and a car
travelling west.
2. Unknown: The rate of change of the distance between the two cars.
3. Diagram:
Notation:
x is the position of the car moving south.
y is the position of the car moving west.
dx/dt = 25 mph.
dy/dt = 15 mph.
We want to find dz/dt.
4. The formula relating the quantities is (from the diagram): + = . Nothing needs to be
eliminated.
5. Implicitly differentiate to introduce derivatives.
+ =
( + = )
2 + 2 =2
6. Solve the unknown rate
= ( + )
7. Substitute values to determine the unknown values
After 2 hrs:
x=25mph(2hrs)=50miles
y=15mph(2hrs)=30miles
= + = √50 + 30 = 10√34 Miles
= ( + )
= (50() + 30(15))
√
= 5√34
124 | P a g e
Debre Tabor Univrsity Applied Mathematics II
Prepared by: Dagnaw Tantie
8. After two hrs, the distance between the two cars is increasing at the rate of 5√34~29.15
mps.
Example 2: A spotlight on the ground shines on a wall 12m away. A man 2m tall walks from the
spotlight towards the building at a speed of 1.6m/s. How fast is the length of his shadow on the
building decreasing when he is 4m from the building?
1. We are given the speed of the man.
2. We want to find the rate of change of the length of his shadow.
3. Diagram:
Notation:
x is the man’s distance from the wall.
h is the height of the shadow.
The height of the man is 12m.
dx/dt = −1.6 m/s (negative since it is decreasing).
dh/dt is what we want to find.
4. Similar triangles:
Geluty
L'Hopitals Rule
125 | P a g e
Debre Tabor Univrsity Applied Mathematics II
Prepared by: Dagnaw Tantie
Let f and g be functions that are differentiable on an interval ( , ) containing c
( )
except possible c itself. Assume that ′( ) ≠ 0. If lim → produce the
( )
( ) ( )
indeterminate form, then lim → = lim → .
( ) ( )
, , ∞ , . ∞,
Examples:
1. lim → = = lim → =2 = 2.
2. lim → = = lim → = 0.
= lim → ( )
= lim → = .
Types of indeterminate
Example: Find
b. lim → e. lim →
Solutions:
126 | P a g e
Debre Tabor Univrsity Applied Mathematics II
Prepared by: Dagnaw Tantie
a. lim → = = lim → = .
b. lim → = = lim → = 0.
c. lim → = = lim → = .
.
e. lim → = = lim → =
. . .
= lim → = .
Example : Find
a lim → c lim → √
( )
b lim → d lim →
Solutions:
a. lim → = = lim → = .
. . .
b. lim → = = lim → = = lim → = ∞.
√ √
c. lim → = lim → = lim → = = lim → = 0.
√
√
( ) ( ).
d. lim → = = lim → = lim →
= = lim → = lim → = 0.
127 | P a g e
Debre Tabor Univrsity Applied Mathematics II
Prepared by: Dagnaw Tantie
( )
If ( ). ( ) is of the form 0. ∞, we can write as ( ). ( ) = or
( )
( )
( ). ( ) = .
( )
Examples: Find
a. lim → sin ln
b. lim → √
Solution:
= lim → = lim →
. . .
2
1
= lim + = lim + − sin = 0
→0 −x.sin1 .cos
sin
→0 cos 0
× ×
lim → − = = 0.
√ √
b. lim → √ = [0 ∗ ∞] = lim → = = lim →
= lim → = 0.
√
Example : Find
128 | P a g e
Debre Tabor Univrsity Applied Mathematics II
Prepared by: Dagnaw Tantie
a. lim → (csc − cot = [∞ − ∞] = lim → −
= lim → ( )= .
= lim → ( )= =0.
b. lim → (x − ln ) = [∞ − ∞] = lim → 1−
= lim → (1) = ∞
= ln lim .
→
= ln lim = ∞.
→
Examples: Find
a. lim → 1+
b. lim →
Solution:
a. lim → 1+ = lim →
( ) ( )
→
= lim → =
129 | P a g e
Debre Tabor Univrsity Applied Mathematics II
Prepared by: Dagnaw Tantie
.( ) .( )
( )
Now lim = lim = lim = lim = 1.
→ → → →
( )
→
Thus lim → 1+ = = = .
( ) ( )
b. lim → = lim → = →
= lim ( )( )
= = lim ( ) ( )
→ →
= lim = −1.
→
( )
Thus lim → = → = = .
6. Indeterminate forms of type
Example : Find
A. lim →
B. lim → (sin 2 )
Solutions:
A. lim → = = lim → = → .
Now lim ln = [0. −∞] = lim = = lim = lim − =0
→ → → →
Thus lim → = → = =1
B. lim → (sin 2 ) =
. ( ) . ( )
= lim → = →
( )
Now, lim tan 3 . ln(sin 2 ) = [0. ∞] = lim =
→ →
130 | P a g e
Debre Tabor Univrsity Applied Mathematics II
Prepared by: Dagnaw Tantie
.( )
= lim = lim =
→ →
= lim . sin 3 . sin 3
→
= lim . lim cos 2 . sin 3
→ →
= × × 0 = 0.
. ( )
(sin 2 )
Thus, lim → = → = =1
7. Indeterminate forms of type ∞
.
Solution: lim → = lim → = lim → = → .
Now, lim = = lim = 0.
→ →
Thus lim → = → = = 1.
131 | P a g e
Debre Tabor Univrsity Applied Mathematics II
Prepared by: Dagnaw Tantie
Unit 5
Integration
a) ( )= b) ( ) = cos
Remark: if F(x) is the anti-derivative of f on some open interval I, the most general anti-derivative of f is
F(x)+c.
a) ( )=
b) ( )=
c) ( )=0
d) ( )=
e) ( )=
4.2 Indefinite integral and their properties
Definition: If ( ) is any anti-derivative of f(x) then the most general anti-derivative of f(x) is called an
indefinite integral and denoted by ∫ ( ) = ( ) + , Where C is any constant.
Note: In the above definition the ∫ is called the integral symbol, f(x) is called the integrand, x is called
the integration variable and c is called the constant of integration.
Examples:
132 | P a g e
Debre Tabor Univrsity Applied Mathematics II
Prepared by: Dagnaw Tantie
b. Since (2 + 1) = 6(2 + 1) , ∫ 6(2 + 1) = (2 + 1) + .
Theorem: If G(x) and F(x) are anti-derivatives of the function f(x) in the interval [ , ] then ( )=
( ) + , for all x in [ , ], where c is arbitrary constant.
So ( ) − ( ) = ⇒ ( )= ( )+
Example: The function ( ) = sin and ( ) = sin + 2 are both anti-derivatives of the function
( ) = cos . Clearly ( ) = ( ) + 2.
Proof: Let and be any real numbers in ( , ) with < . Since f is differentiable on ( , ), it
must be differentiable on ( , ) and [ , ]. Therefore we can apply the MVT to f on the interval
[ , ] by which there exists a number c in ( , ) such that
( )− ( ) = ( )( − ) = 0( − )=0
⇒ ( ) = ( ), ∀ , ∈ ( , ).
Thus f is constant on ( , ).
Note: The process of finding the indefinite integral is called integration or integrating f(x).
1) ∫ = +
2) ∫ = + , ≠ −1.
3) ∫ = +
4) ∫ = ln| | +
5) ∫ = + .
6) ∫ sin = − cos +
7) ∫ cos = sin + .
133 | P a g e
Debre Tabor Univrsity Applied Mathematics II
Prepared by: Dagnaw Tantie
8) ∫ sec = tan + .
9) ∫ sec tan = sec + .
10) ∫ csc = − cot + .
11) ∫ csc cot = −csc + .
1) ∫ kf( ) = ∫ f( ) .
2) ∫ (f( ) ± ( )) = ∫ f( ) ± ∫ g( ) .
a) ∫(4 + 2 sin ) .
b) ∫(3 −2 + 7 + 2) .
c) ∫( + 3) ( + 1) .
d) ∫(6 − 10 sec + 5 cos ) .
e) ∫ .
Solution:
b) ∫(3 −2 + 7 + 2) = − + +2 + .
c) ∫( + 3) ( + 1) = ∫( + 6 + 9) ( + 1)
= ∫( +6 +9 + + 6 + 9) .
= ∫( +7 + 15 + 9) .
= + + +9 + .
e) ∫ =∫ −1 = ∫ 7 csc −1 = −7 cot − + .
Exercise: Falling object has acceleration a(t) =-32; and the position s(t) at time t if s(0) = 1000
and the initial velocity is -20. When is s(t) = 0?
5.2. Partitions, Lower sum, Upper sum, Riemann sum
I. Partitions
134 | P a g e
Debre Tabor Univrsity Applied Mathematics II
Prepared by: Dagnaw Tantie
Definition: A partition of an interval [a, b] is a finite set ={ , , ,…, , }, where
= < < <⋯< < = .
The points in P divide [a, b] into subintervals [ − ], = 1, 2, … , , whose lengths are
denoted by ∆ = − .
The maximum of the length of the subintervals of the partition P; i.e. Max {∆ : = 1,2, … , } is
called the norm or size or mesh of P and it is denote by ‖ ‖ or ∆ .
II. Lower sum and Upper sum
Definition: Let ={ , , ,…, , } be partition of the closed interval [a, b] and f be
abounded function defined on that interval. Then
a) the lower sum of f with respect to the partition P is defined as:
( )=∑ ( − )=∑ ∆ . Where is the infimum of f in the
interval [ , ]. i.e. = { ( ): ≤ ≤ }.
b) the upper sum of f with respect to the partition P is defined as:
( )=∑ ( − )=∑ ∆ . Where is the supermum of f in the
interval [ , ]. i.e. = { ( ): ≤ ≤ }.
0, , 1, , 2 .
Solution: The subintervals associated with partition p are 0, , , 1 , 1, and , 2 and the
minimum value for each subinterval associated with P is attained at the left end point of
subinterval.
135 | P a g e
Debre Tabor Univrsity Applied Mathematics II
Prepared by: Dagnaw Tantie
Thus, = (0) = 0, = = , = (1) = 1, = = .
( )=∑ ∆ = ∆ + ∆ + ∆ + ∆
= 0. + . + 1. + . = .
Also the maximum value of f on each subinterval associated with P is attained at the right end
point of subintervals:
We have, ∆ =∆ = ∆ =∆ = .
Therefore, ( )=∑ ∆ = ∆ + ∆ + ∆ + ∆ .
= . + 1. + . + 2. = .
Note:
( )≤ ( ) and ( )≤ ℎ ≤ ( ).
Equal width partition are called regular partitions. The formula for point in a regular
partition is = + ∆ .
Example 2: Let ( ) = + 1 on [0, 2]. Determine the upper and lower Riemann sums for a
So ∆ = = = .
= + ∆ = 0+ = .
Step 3: Take a look at the graph of ( ) = + 1 on [0, 2]. Since f is an increasing function,
the maximum value of f on each subinterval occurs at the right hand of the interval. The right
hand of the interval is just .
Consequently, the maximum value of f on the interval is
= ( )= =1+ ( ) = + 1.
136 | P a g e
Debre Tabor Univrsity Applied Mathematics II
Prepared by: Dagnaw Tantie
Step 4: putting this all together, the upper sum is
( )=∑ ∆ =∑ ( )∆ .
=∑ . .
=∑ +1 . .
=∑ + .
=∑ +∑ .
( )( )
= ∑ +∑ . Since ∑ = .
( )( )
= + (4(1))
= .
The lower sum can be calculated in a similar way. Again, because the function is increasing the
minimum value of f on the sub interval curs at the left hand end point .
Consequently the minimum value of f on the interval is
= ( )= =1+ ( ) = − + .
=∑ . .
=∑ − + . .
=∑ + .
=∑ − ∑ + ∑ .
( )( ) ( )
= − ( ) + (4 )
= .
137 | P a g e
Debre Tabor Univrsity Applied Mathematics II
Prepared by: Dagnaw Tantie
Example 3: Let ( ) = + 1 on [0, 2]. Determine the upper and lower Riemann sums for a
So ∆ = = = .
= + ∆ = 0+ = .
Step 3: Take a look at the graph of ( ) = + 1 on [0, 2]. Since f is an increasing function,
the maximum value of f on each subinterval occurs at the right hand of the interval. The right
hand of the interval is just .
Consequently, the maximum value of f on the interval is
= ( )= 2 = 1+ ( ) = + 1.
=∑ . .
=∑ +1 . .
=∑ + .
=∑ +∑ .
= ∑ + ∑ 1.
( )( )
= . + (1) .
( )( )
= . + 2.
( )
= . + 2.
= . (2 + + ) + 2.
= + + .
138 | P a g e
Debre Tabor Univrsity Applied Mathematics II
Prepared by: Dagnaw Tantie
The lower sum can be calculated in a similar way. Again, because the function is increasing the
minimum value of f on the sub interval curs at the left hand end point .
( )
Thus, = is the point at which f has minimum value on the interval [ , ].
= ∑ 1+ ∑ − ∑ + ∑ 1.
( )( ) ( )
= ( )+ . − . + .
( )( ) ( )
= 2+ . − + .
10 2 2
= − + .
3 3
c) Riemann Sum
DEFINITION(Riemann Sum): Suppose f is defined on the interval [a, b] with partition
= < < <⋯< < = . Let ∆ = − and let be any point
chosen so that ≤ ≤ . Then
∑ ( )∆ is called a Riemann sum for f on [a, b].
Note: In the general defnition of Riemann sum we have not assumed that f is non-negative or that it
is continuous.
Example: Let ( ) = 5 + 1 and consider the partition of the interval [−2, 1] by means of the set of
partition points −2, − , −1, −0.3, 0.2, 1 . Find the Riemann sum of f.
139 | P a g e
Debre Tabor Univrsity Applied Mathematics II
Prepared by: Dagnaw Tantie
Solution: The intervals associated with partition P are
∆ = −1 − (−1.5) = 0.5
∆ = 1 − (0.2) = 0.8
suppose we choose
∗
= −1.8
∗
= −1.2
∗
= −0.3
∗
= 0.2
∗
=1
∑ ∗ ∗) ∗) ∗) ∗) ∗)
( )∆ = ( ∆ + ( ∆ + ( ∆ + ( ∆ + ( ∆
∗) ∗) ∗) ∗) ∗)
= ( ∆ + ( ∆ + ( ∆ + ( ∆ + ( ∆
= −2.75.
140 | P a g e
Debre Tabor Univrsity Applied Mathematics II
Prepared by: Dagnaw Tantie
Note: In the above example ;
Note:
a. ∫ 2
b. ∫ ( + 2)
c. ∫
Solution (a): The function ( ) = 2 is integral on [1, 4] because it is continuous on [1, ] . Now
divide into n subintervals of equal width.
∆ =∆ = = = .
∗
Choosing as the right end point of each subinterval produces
∗
= + (∆ ) = 1 + .
141 | P a g e
Debre Tabor Univrsity Applied Mathematics II
Prepared by: Dagnaw Tantie
Therefore the indefinite integral is given by
∑ ∗
∫ 2 = lim → ( )∆ .
= lim → ∑ (1 + )( )
= lim → ∑ 1.
= lim → . =6
∆ =∆ = = = .
∗
Choosing as the right end point of each subinterval produces
∗
= + (∆ ) = 0 + = .
∑ ∗
∫ = lim → ( )∆ .
= lim → ∑ ( )( )
= lim → ∑ ( )
= lim → ∑ .
( )( )
= lim → 8(
( )( )
= lim → 8( = .
142 | P a g e
Debre Tabor Univrsity Applied Mathematics II
Prepared by: Dagnaw Tantie
If f is continuous and nonnegative on the closed interval [ , ] , then the area of the region
bounded by the graph of f, the x-axis, and the vertical line = and = is given by
=∫ ( )
1) ∫ f(x)dx = − ∫ f(x)dx.
2) ∫ f(x)dx = 0.
3) ∫ k. f(x)dx = k. ∫ f(x)dx
6) ∫ f(x)dx = ∫ f(t)dt
i. even if (− ) = ( ).
ii. odd if (− ) = − ( ).
a) ∫ f(x) dx.
b) ∫ (2f(x) − 3h(x))dx.
c) ∫ f(x) dx.
143 | P a g e
Debre Tabor Univrsity Applied Mathematics II
Prepared by: Dagnaw Tantie
Solution:
= 2∫ f(x)dx − 3 ∫ h(x)dx
= −2 ∫ f(x)dx + 3 ∫ h(x)dx
= −2(5) + 3(4) = 2.
a) ∫ ( ) =∫ ( ) −∫ ( ) =
b) ∫ ( ) =∫ ( ) +∫ ( ) =
c) ∫ ( ) = −(∫ ( ) +∫ ( ) )=−
a) ( )=∫ 3 .
b) ( ) = ∫ cos + sin .
c) ( ) = ∫ √1 +
Remarks
( )
a) ∫ ( ) = ′( ) ( ( )).
b) ∫( )
( ) = − ′( ) ( ( )).
( )
c) ∫( )
( ) = ′( ) ( ( )) − ′( ) ( ( )).
Example:
a) ( ) = (∫ ).
144 | P a g e
Debre Tabor Univrsity Applied Mathematics II
Prepared by: Dagnaw Tantie
b) ( ) = (∫ (sin ) ).
c) ( ) = ∫ √1 + .
and ( ) = ′( ), then ∫ ( ) = ( ) − ( ).
d) ∫ 3 .
e) ∫ (sin + 3 cos ) .
f) ∫ |2 − 1| .
−(2 − 1) <
c: We know that |2 − 1| = and hence
(2 − 1) >
∫ |2 − 1 | = ∫ −(2 − 1) +∫ 2 −1 = (− + )| + ( − )| = .
( )
Note: If ( ) = ∫ () , then ( ) = ( ) . ′ ( ).
Example: Find the derivative of the following functions, using the fundamental theorem of
calculus.
a) (∫ cos + sin ).
b) (∫ ).
145 | P a g e
Debre Tabor Univrsity Applied Mathematics II
Prepared by: Dagnaw Tantie
c) (∫ (sin ) ).
Solution:
Average of integral
Suppose f is a continuous function on [a, b], then there is a number ∈ [ , ] such that
( )= ∫ ( ) .
5.6.Techniques of integration
I. Integration by Substitution
Theorem: Let f and g be functions with both and ′ continuous on an interval I. If G is anti-
derivative of g, then ∫ ( ) ( ) = ( ) + … … … … … . (∗)
Note:
Examples: Evaluate
a) ∫ (8 − 5) f) ∫ sin cos k) ∫
√
√
b) ∫( + 1) 2 g) ∫ √ −1 .
l) ∫
c) ∫ sin 3 h) ∫ cos
m) ∫
d) ∫ i) ∫ n)
( )
e) ∫ √ −1 . j) ∫
Solution:
a) Let = 8 −5 ⇒ =8 ⇒ = . Thus
( )
∫ (8 − 5) =∫ = ∫ = . + = + = + .
146 | P a g e
Debre Tabor Univrsity Applied Mathematics II
Prepared by: Dagnaw Tantie
b) Let = +1 ⇒ =2 Thus
∫( + 1) 2 =∫ = + = + .
c) Let =3 ⇒ =3 ⇒ = . So that
∫ =∫ . = ∫ = ∫ = + = −√ + = −√2 − + .
( )
sin − + .
i) Letting = ln ⇒ = Thus
∫ =∫ . =∫ . = ln + = ln(ln ) +
j) Letting = ln ⇒ = Thus
( )
∫ =∫ = + = + .
k) Letting =√ ⇒ = ⇒ =2 .Thus
√ √
147 | P a g e
Debre Tabor Univrsity Applied Mathematics II
Prepared by: Dagnaw Tantie
√
∫ = ∫ cos . 2 = 2 sin + = 2 sin√ + .
√
a) ∫ .
b) ∫ .
c) ∫ .
∫ ( ) ( ) = ( ) ( )−∫ ( ) ( ) .
Examples: Evaluate
a) ∫ d) ∫
b) ∫ sin 2 e) ∫ cos
c) ∫ ln f) ∫ cos
Solution:
sin 2 + .
∫ ln = ln − ∫ . = ln − ∫ = ln − + .
148 | P a g e
Debre Tabor Univrsity Applied Mathematics II
Prepared by: Dagnaw Tantie
∫ =− +∫ .2
=− + 2 (− )− ∫− .2
=− + 2 (− )+ 2∫
=− + 2 (− )−2 + .
e) Letting = and = cos implies = and = sin . Thus
∫ cos = sin − ∫ sin . Again letting = and = sin implies
= and = − cos . Thus,
∫ cos = sin − ∫ sin
= sin − (− cos ) − ∫ (−cos ) .
= sin + (cos ) − ∫ (cos ) .
⇒∫ cos +∫ cos = sin + (cos ).
⇒ 2∫ cos = sin + (cos ).
To find the integral we apply integration by part. Letting = and = cos implies =
and = sin . Thus
( )
The rational function R(x) is a quotient of two polynomials. That is, ( )= . The rational
( )
i. If the degree of the numerator is less than the degree of the denominator then f is called a
proper rational.
149 | P a g e
Debre Tabor Univrsity Applied Mathematics II
Prepared by: Dagnaw Tantie
ii. If the degree of the numerator is greater than or equal to the degree of the denominator
then f is called improper rational.
iii. The process( a systematic method) of integrating rational function is known as partial
fraction method.
Case I: If you have an improper rational function (the degree of the numerator is equal to or
greater than the degree of the denominator) the preliminary step of long division is necessary.
For instance ∫ is improper integral. So by long division we have =2− and
hence ∫ = ∫ 2− = 2 + ln(2 + 5) +
Case II: If you have a proper rational function (the degree of the numerator is less
than the degree of the denominator) then you are ready to proceed to the step of partial
fractions.
Theorem: Any polynomial with real coefficients can be expressed as linear and quadratic factors
in such a way that each of the factors has real coefficients.
3 Q(x) contains irreducible quadratic factors, none of corresponding partial fractions of the
which is repeated. form
Suppose ( ) = + + . ( )
= .
( )
4 Q(x) contains a repeated irreducible quadratic factor corresponding partial fractions of the
Suppose ( ) = ( + + ) . form
150 | P a g e
Debre Tabor Univrsity Applied Mathematics II
Prepared by: Dagnaw Tantie
( )
= +( )
+ ⋯+
( )
( )
Examples: Evaluate
a) ∫ .
b) ∫ ( )(
.
)
c) ∫ .
d) ∫ ( )(
.
)
e) ∫ ( )
f) ∫ ( )
Solution:
a) We have =( )(
. we obtain
)
( ) ( )
( )(
= + = ( )(
.
) )
If = 3 then 9 = (3 + 4) ⇒ 9=7 ⇒ = .
If = −4 then 9 = −7 ⇒ =− .
Thus ∫ =∫ ( )( )
=∫ ( )
+ ( )
=∫ ( )
+∫ ( )
= ∫ ( )
+ ∫ ( )
= ln| + 4| + ln| − 3| + .
151 | P a g e
Debre Tabor Univrsity Applied Mathematics II
Prepared by: Dagnaw Tantie
b) We obtain A, B, and C such that
c) ∫ .
d) ∫ ( )(
.
)
e) ∫ ( )
f) ∫ ( )
Trigonometric Integrals
3. ∫ tan = + ∫ tan ,
4. ∫ sec = + ∫ sec , ≠1
5. ∫ csc =− + ∫ csc , ≠1
6. ∫ tan = + ∫ tan ,
a) ∫ cos b) ∫ tan
1. Strategy for Evaluating integrals of the form ∫
a) If the power cos is odd (n=2k+1), save one cosine factor and express the remaining
factors in terms of sine using trig identity. Then substitute = sin .
b) If the power sin is odd (m=2k+1), save one sine factor and express the remaining
factors in terms of cosine using trig identity. Then substitute = cos .
Note that if the powers of both sine and cosine are odd, either (a) or (b) can be used.
152 | P a g e
Debre Tabor Univrsity Applied Mathematics II
Prepared by: Dagnaw Tantie
c) If the powers of both sine and cosine are even, use the half-angle identities, sin =
(1 − cos 2 ) and cos = (1 + cos 2 ). It is sometimes helpful to use the identity,
a) If the power of cosecant is even (n=2k, k≥2), save a factor of and express the
remaining factors in terms of cot using identity. Then substitute cot .
b) If the power of cotangent is odd (n=2k+1), save a factor of csc cot and express the
remaining factors in terms of csc . The substitute csc .
4. Strategy for Evaluating integrals of the form (a) ∫ ,
(b) ∫ , or (c) ∫ , use the following identity:
Trigonometric Substitutions
There are three kinds of trig subs. You use them when you see as part of the integrand one of the
expressions √ − , √ + ,or √ − , where a is some constant . The
substitution will simplify the integrand since it will eliminate the square root. Here's a table
summarizing the substitution to make in each of the three kinds.
Expressions Substitution Relation between Substitution implies
and
153 | P a g e
Debre Tabor Univrsity Applied Mathematics II
Prepared by: Dagnaw Tantie
a a
− = sin = cos − = cos
b b
a a
+ = tan = sec u + = sec
b b
a a
− = sec = secu − = tan
b b
There are also right triangles you can draw to make the connections between x, a, and . The
three triangles below refer to the three trig subs, respectively.
Exercise
1. Suppose that, (1) = 2, (4) = 7, ′(1) = 5, and is continuous. Find the value of
∫ ′′( ) .
2. A particle moves on a straight line with velocity function ( ) = sin cos . Find its
position function s=f(t) if f(0)=0.
3.
a) ∫ √
b) ∫ √
c) ∫ √
Solution:
a) Substitute = 4 sin , so that = cos .Then
∫ =∫ 4cos =∫ 4cos
√ √ .
=∫ = ∫ csc =− cot +
√
=− +
b) ∫ √ = ∫√ 4 cos = ∫ 16 = 16 ∫( ) .
= 16 ∫ − = 16 ∫ −
= 8 tan − . √16 − + .
∫√ =∫ = ∫
√
= ∫ cos = ∫ = ∫1 = +
√ √
( ( ) )
√ √
√
= sin ( )+
Solution:
a) We have ∫ √ =∫ =∫
( ) ( )
∫√ =∫ =∫ sec = ∫ n
( ) ( )
Integrals containing √ − .
a) ∫ √
b) ∫ √1 +
5.7. Improper integrals
Definition: An integral is an improper integral if either the interval of integration is not finite
(improper integral of type 1) or if the function to integrate is not continuous (not bounded) in the
interval of integration (improper integral of type 2).
Example:
∞
1. ∫ is an improper integral of type 1 since the upper limit of integration is infinite.
2. ∫ is an improper integral of type 2 because is not continuous at 0.
∞
3. ∫ is an improper integral of types 1 since the upper limit of integration is infinite. It is
also an improper integral of type 2 because is not continuous at 1 and 1 is in the interval
of integration.
156 | P a g e
Debre Tabor Univrsity Applied Mathematics II
Prepared by: Dagnaw Tantie
5. ∫ tan is an improper integral of type 2 because tan x is not continuous at .
Improper integrals of Type I (improper integrals with infinite integration limits)
In this case if the limit is finite we say that the improper integrals converges and that the limit is
the value of the improper integral. If the limit doesn't exist, the improper integral diverges.
a) ∫
b) ∫
c) ∫
d) ∫
e) ∫
f) ∫ (1 − )
g) ∫ .
Solution:
a) ∫ =
157 | P a g e
Debre Tabor Univrsity Applied Mathematics II
Prepared by: Dagnaw Tantie
Improper integral of type 2
Definition: Improper integrals of type 2 are evaluated as follows:
1. iI f is continuous on [a, b) and not continuous at b then we define
∫ ( ) = lim → ∫ ( ) .
2. If f is continuous on (a, b] and not continuous at a then we define
∫ ( ) = lim → ∫ ( ) .
3. If f is not continuous at c and f is continuous on [ , ) ∪ ( , ], where a < c < b, then
∫ ( ) =∫ ( ) +∫ ( )
A Comparison Test for Improper Integrals
Comparison test: Suppose that f and g are continuous functions with ( ) ≥ ( ) ≥ 0 for
≥ .
∞ ∞
4. If ∫ ( ) is convergent, then ∫ ( ) is convergent.
∞ ∞
5. If ∫ ( ) is divergent, then ∫ ( ) is divergent.
∞ ∞
Example1: The integral ∫ is convergent, because > > 0 and ∫ is convergent
by the p-test, since = 2 > 1.
∞
Example 2: Does the integral ∫ converge?
Solution: We have, 0 < < .
∞ ∞ ∞
Note that ∫ is convergent, since ∫ =∫ = lim →∞ ∫ = .Therefore the
∞
integral∫ converges.
∞
Example 3: Does the integral ∫ converge?
√
158 | P a g e
Debre Tabor Univrsity Applied Mathematics II
Prepared by: Dagnaw Tantie
5.8. Application of integration
a) Area
Definition: The area A of the region bounded by the curves = ( ), = ( ), and the lines
= & = , where f and g area continuous and ( ) ≥ ( ) for all x in [ , ] is
∫ ( )− ( ) .
Example 1: Find the area of the region bounded above by = 2 + 3, bonded below by = ,
and bounded on the sides by the lines = 0 and = 2.
Solution: The upper boundary curve is = 2 + 3 and the lower boundary curve is = .
So letting ( ) = 2 + 3 , ( ) = 2 + 3,
= 0 and = 2 gives
∫ (2 + 3 − ) =( +3 − )
= 2 + 3(2) − −0= .
Example 2: Find the area of the region that is enclosed between the curves = and =2 +
3.
Solution : Sketching these two curves shows that the lower boundary is = and the upper
boundary = 2 + 3.
At the endpoints of the region, the upper and lower boundaries have the same y-coordinates;
thus, to find the endpoints we equate
= and = 2 + 3.
This yields
=2 +3 ⇒ −2 −3 =0 ⇒
( − 1)( + 3) = 0.
From which we obtain = 3 an = −1.
159 | P a g e
Debre Tabor Univrsity Applied Mathematics II
Prepared by: Dagnaw Tantie
Thus the points of intersection are (−1, 1) and (3, 9). This implies the region lies between
= −1 and = 3. So
∫ (2 + 3 − ) =( +3 − )
( )
= 3 + 3(3) − − (−1) + 3(−1) − =9+ = .
Example 3: Find the area of the region bounded by the curves = sin , = cos , =
0 = .
Solution: The points of intersection occur when sin = cos . i.e. when = . Since 0 ≤ ≤ .
= (sin + )| + (− cos − )|
(− cos − )
= 2√2 − 2 .
b) Volume
Definition: The volume of a sold of known integrable cross-sectional area ( ) from = to
= is the integral of A from a to b,
=∫ ( ) .
Note:
160 | P a g e
Debre Tabor Univrsity Applied Mathematics II
Prepared by: Dagnaw Tantie
1. This definition applies whenever A(x) is continuous, or more generally, when it is
integrable.
2. A cross-section of a solid S is the plane region formed by intersecting S with a plane.
3. If the cylindrical solid has a known base area A and height h, then the volume of the
cylindrical solid is
= .ℎ ℎ
To apply the formula in the definition to calculate the volume of a solid, take the following steps:
1) Sketch the solid and a typical cross-section.
2) Find a formula for A(x), the area of a typical cross-section.
3) Find the limits of integration.
4) Integrate A(x) using the Fundamental Theorem.
Example; A 3m high pyramid has a square base that is 3 m on a side. The cross-section of the
pyramid perpendicular to the altitude x m down from the vertex is a square x m on a side. Find
the volume of the pyramid.
Solution
1. A sketch. We draw the pyramid with its altitude along the x-axis and its vertex at the
origin and include a typical cross-section.
161 | P a g e
Debre Tabor Univrsity Applied Mathematics II
Prepared by: Dagnaw Tantie
2. A formula for A(x). The cross-section at x is a square x meters on a side, so its area is
( )= .
3. The limits of integration. The squares lie on the planes from x=0 to x=3.
4. Integrate to find the volume.
=∫ ( ) =∫ =( ) =9 .
Solids of Revolution
A solid of revolution is a solid that is generated by revolving a plane region about a line that
lies in the same plane as the region; the line is called the axis of revolution.
When calculating the volume of a solid generated by revolving a region bounded by a given
function about an axis, follow the steps below:
1. Sketch the area and determine the axis of revolution, (this determines the variable of
integration).
2. Sketch the cross-section, (disk, shell, washer) and determine the appropriate formula.
3. Determine the boundaries of the solid,
4. Set up the definite integral, and integrate.
162 | P a g e
Debre Tabor Univrsity Applied Mathematics II
Prepared by: Dagnaw Tantie
i. Finding volume of a solid of revolution using a disc method.
The simplest solid of revolution is a right circular cylinder which is formed by revolving a
rectangle about an axis adjacent to one side of the rectangle, (the disc).
Definition:
1) The volume of the solid generated by a region under f(x) bounded by the x-axis and
vertical lines x=a and x=b, which is revolved about the x-axis is
2) The volume of the solid generated by a region under ( ) (to the left of ( ) bounded by
the y-axis, and horizontal lines y=c and y=d which is revolved about the y-axis.
Example 1: Find the volume of the solid obtained when the region under the curve = √ over
the interval [1, 4] is revolved about the x-axis.
Solution: The volume is
=∫ ( ( )) =∫ = .( ) =8 − = .
163 | P a g e
Debre Tabor Univrsity Applied Mathematics II
Prepared by: Dagnaw Tantie
Example 2: Derive the formula for the volume of a sphere of radius r.
Solution: As indicated in Figure below, a sphere of radius r can be generated by revolving
the upper semicircular disk enclosed between the x-axis and + = about the x-axis.
Since the upper half of this circle is the graph of = ( ) =√ − , it follows that the
volume of the sphere is
= ( ) = ( − ) = . −
3
= − − − + .
= − + − .
=2 − .
=2 = .
164 | P a g e
Debre Tabor Univrsity Applied Mathematics II
Prepared by: Dagnaw Tantie
=∫ |( ( )) − ( ( )) | (washer with respect to x)
2) The volume of the solid generated by a region between ( ) and ( ) bounded by the
horizontal lines = and = which is revolved about the y-axis.
The volume V of a cylindrical shell with inner radius , outer radius , and height h can be
written as
165 | P a g e
Debre Tabor Univrsity Applied Mathematics II
Prepared by: Dagnaw Tantie
= [ ] • [ℎ ℎ ]
=[ − ]ℎ
= [ + ][ − ]ℎ
=2 . ℎ. [ − ]
= 2 • [ ] • [ℎ ℎ] • [ ℎ ].
The volume V of the solid can be obtained by adding together the volumes of the tubes; that
is, = ( ) + ( ) + ⋯ + ( ).
Definition:
a. The volume of the solid generated by revolving about the y-axis the region between the
x-axis and the graph of a continuous function y = f (x), ≤ ≤ is
=∫ 2 ( )
b. The volume of the solid generated by revolving about the x-axis the region between the
y-axis and the graph of a continuous function = ( ), ≤ ≤ is
=∫ 2 ( )
166 | P a g e
Debre Tabor Univrsity Applied Mathematics II
Prepared by: Dagnaw Tantie
Example 1: Use cylindrical shells to find the volume of the solid generated when the region
enclosed between = √ , = 1, = 4, and the x-axis is revolved about the y-axis.
Solution: First sketch the region (Figure a below); then imagine revolving it about the
y-axis (Figure b below). Since ( )= √ , = 1, and = 4, definition (1) yields
=∫ 2 √ =2 ∫ = 2 . = (32 − 1) = .
Example 2: Use cylindrical shells to find the volume of the solid generated when the region R in
the first quadrant enclosed between = and = is revolved about the y-axis (Figure a
below).
Solution. As illustrated in part (b) of Figure below, at each x in [0, 1] the cross section of
R parallel to the y-axis generates a cylindrical surface of height − and radius x. Since
the area of this surface is 2 ( − ).
Thus the volume of the solid is
=∫ 2 ( − ) =2 ∫ ( − ) =2 − =2 − = .
167 | P a g e
Debre Tabor Univrsity Applied Mathematics II
Prepared by: Dagnaw Tantie
Example 3: Use cylindrical shells to find the volume of the solid generated when the region R
under = over the interval [0, 2] is revolved about the line = −1.
Solution. First draw the axis of revolution; then imagine revolving the region about the axis
(Figure a below As illustrated in Figure b below, at each y in the interval 0 ≤ ≤ 4, the cross
section of R parallel to the x-axis generates a cylindrical surface of height 2 − and radius
+ 1 . Since the area of this surface is
2 ( + 1)(2 − )
it follows that the volume of the solid is
= 2 ( + 1) 2 − =2 2 − +2−
=2 − +2 − = .
168 | P a g e
Debre Tabor Univrsity Applied Mathematics II
Prepared by: Dagnaw Tantie
6.3. Arc Length
Definition 1: If ƒ is continuously differentiable on the closed interval [a, b], the length of the
curve (graph) y=f(x) from x=a to x=b is
=∫ 1+( ) =∫ 1 + ( ′( )) .
Definition 2: If g is continuously differentiable on the closed interval [c, d], the length of the
curve (graph) x=f(y) from x=c to x=d is
=∫ 1+( ) =∫ 1 + ( ′( )) .
b) = ln (sec ) ,0 ≤ ≤ .
6.4.Surface Area
Surface area for revolution about the x-axis
Definition: If the function ( ) ≥ 0 is continuously differentiable on [a, b], the area of the
surface generated by revolving the curve = ( ) about the x-axis is
=∫ 2 1+( ) =∫ 2 ( ) 1 + ( ′( )) .
Example 1: Find the area of the surface generated by revolving the curve about the x-axis
= 2√ , 1 ≤ ≤ 2.
=∫ 2 1+( )
= ∫ 2 2√ 1+( )
√
=4 ∫ √ 1+
=4 ∫ √
√
=4 ∫ √ =4 ∫ √ +1
√
= 4 . ( + 1)
= 4 . ((3 + 1) − ((1 + 1) )
169 | P a g e
Debre Tabor Univrsity Applied Mathematics II
Prepared by: Dagnaw Tantie
= (3√3 − 2√2)
170 | P a g e
Debre Tabor Univrsity Applied Mathematics II
Prepared by: Dagnaw Tantie